Showing posts with label MCCQE. Show all posts
Showing posts with label MCCQE. Show all posts

11.11.11

DOWNLOAD TORONTO NOTES 2010 FREE



DOWNLOAD TORONTO NOTES 2010 FREE

CLICK HERE

FREE MCCQE SAMPLE QUESTIONS

CLICK DOWN TO GO TO SITE WITH FREE MCCQE SAMPLE QUESTIONS

CLICK HERE

Infectious Disease MCQ MCCQE


                                          Infectious Disease MCQ

1-    22 year old male presents to the ER with a 1 day history of erosions of the lips, conjuctivitis and blistering rash. He has a temperature of 37.9 and no other systemic complaints. He is currently on a course of Cotrimoxazole Full dose twice daily PO for an episode of otitis media diagnosed 3 days ago. The best initial step in the management of this patient is:
a.     Prescribe a course of topical steroids
b.     Change antibiotic, preferably to a cephalosporin and send home
c.      Admit to hospital, monitor fluid status and stop antibiotic
d.     Refer to Dermatology for further investigation
e.      Reduce the dose of his antibiotic

2-    A 40 year old diabetic male presents with an acute onset of redness in the hands and forearm with streaks extending up towards the axilla. There is local pain and crepitus. Temp 40, pulse 110 and BP 95/65. WBC is elevated.  The most likely causative offending organism is:
a.     Group A streptococcus
b.     Staph. epidermidis
c.      Viridans Streptoccocus
d.     Group D strep
e.      Group B strep.

3-    In the above patient, the most appropriate approach for therapy is:
a.     IV antibiotic only
b.     Oral Antibiotic only
c.      Surgical debridement with IV antibiotic

Obstetrics MCQ mccqe


                                                Obstetrics MCQ


1-    A 23 year old female who is breastfeeding her 3 week old infant complains about breast pain and fever of 40C. On examination, the left breast is tender with a red and hard right upper quadrant. What is the most appropriate management?
a.     Increase frequency of breastfeeding and analgesia
b.     Perform manual milk extraction for 24 hours
c.      Discontinue breastfeeding, analgesia and antibiotic therapy
d.     Incise and drain the breast
e.      Antibiotic therapy, continue breastfeeding from opposite breast.

2-    A 29 Year old gravida 3 para 3 had a vaginal delivery 30 minutes ago. After delivery of the placenta, she continues to bleed and has now bled approximately 500 mls in the past 15 minutes. The most common cause of this condition is:
a.     Lacerations
b.     Retained placental tissues
c.      Placenta accreta
d.     Uterine Atony
e.      Distended bladder

3-    A 19 year old has chronic asymptomatic hepatitis B infection. She is 22 weeks pregnant. When considering the risk of transmission to her yet to be born baby, you would:
a.     Give hepatitis B immunoglobulin to the mother at 37 weeks
b.     Give the neonate hepatitis B immunoglobulin immediately after birth

OTTAWA NOTES MEGA QUESTIONS

Remembered questions MCCQE CANADIAN BOARD EXAM


________________________________________


>
>PSYCHIATRY
>
>Q1. Depression is associated with which of the following sleep patterns
>-decreased REM latency
>-decreased REM normal NREM
>-decreased stage 2 NREM
>-decreased stage 4 NREM
>- decreased latency REM and decreased 4 stage sleep
>
>Q2.What is the defense mechanism of bipolar personality disorder -Splitting
>
>Q3.Which one of these constitute a part of mature ego defence mechanism
>-Humour
>
>Q4. What is the drug interaction of St-John`s wort with SSRI
>-serotonin syndrome
>-
>
>Q5. An aged female with history of depression for two months comes to
>psychiatrist. She admits using St John¡¦s Wort 300 mg for last 1 week in
>consultation with a naturopath. What should be done?
>-Stop using St John¡¦s Wort and add Sertraline
>-Increase dosage of St John¡¦s Wort to 1800 mg
>-Coadminister St John¡¦s wort at reduced
>dosage of 50% with Sertraline for its safe
>-give paxel
>
>Q6. Pt is on Lithium therapy becomes hypothyroid. What is the treatment?
>-Reduce dose of lithium to 50%
>-Start levothyroxine
>-Stop lithium
>
>Q7. Pt on Lithium therapy became weak, lethargic, and Intolerant to heat.
>What Investigation must be done?
>-Lithium level
>-Monitor TSH level
>
>Q8. In Autistic disorder choose the best option
>-More common in girls
>-Autosomal recessive
>-
>Q9. Handwashing ritual in 11 years old. Mother is very worried and does not
>treatment with any medication. Which one is the best option? which
>psychotherapy?
>-Cognitive behavioral therapy /behavioural therapy
>-Family therapy.
>-biofeed-back
>-psychodynamic psychotherapy
>-relaxation therapy
>
>Q10. Which of the drugs cause amnesia the most?
>-Triazolam
>-Diazepam
>-phenytoin
>
>-Imipramine
>
>Q11. A person is brought into the police station found wandering aimlessly
>and he is unable to tell his name or any other personal information
>regarding time and place. Which of the following would not be part of your
>d/d?
>-Malingering
>-temporal lobe epilepsy
>-dissociative fugue
>-dissociative amnesia (amnestic disorder)
>-factitious disorder
>
>Q12. Which is not a part of normal grief reaction?
>-physical symptoms
>-preoccupation with suicide
>-hearing voices
>-consider himself responsible
>
>Q13. Anti-depressant treatment should be carried on for how many months?
>-2weeks
>-6weeks
>-2years
>-indefinite time
>
>Q14. Paranoia is not seen in
>-pernicious anemia
>-hypothyroidism
>-mania
>
>Q15. Which is true regarding suicide in adolescent?
>-More common in summer
>-Attempt to suicide is
>more common in F than in M.
>-More common in M, ratio 4:1
>-Suicide M:F=10:1 (checked)
>
>Q16. question on Ego dystonic and Ego syntonic
>(NB. Dystonic = OC Disorder D is for D, Syntonic = OC Personality)
>
>Q17. 8 weeks pregnant patient with depression on Sertraline wants to stop
>medication. What will you do?
>-do a detailed physical and mental examination
>-continue antidepressant
>-substitute another
>-lower the dosage
>
>Q18. CBT -goal of therapy should be directed
>
>Q19. BPD -Valproic acid
>
>Q20. clozapine -CBC
>
>Q21. Symptoms of depression improved but mild -continue and inc. the dose
>of flu.
>
>Q22. A patient with schizophrenia & is on treatment. He can not sit still
>(akathisia symptoms). What is the best treatment?
>-Lorazepam
>
>Q23. Dystonia
>-More common in young male (checked)
>-more common in female
>
>
>Q24. 6 yr old child with seizure then followed by paralysis of arm along
>with confusion
>-post ictal Todd¡¦s paralysis
>
>Q25. Characteristics of panic attacks-
>-episodic and symptoms increase in intensity
>-trigger
>
>Q26. TCA side effect -hypotension
>
>Q27. Differentiation b/w Alz. N pseudodementia -MMSE
>
>Q28. Loosening of association A/W
>-schizophrenia
>-mania
>
>Q29 Somatoform disorder- multiple somatic symptoms
>
>Q30. Psycho stimulants are used in which of the following disorders?
>- Attention deficit hyperactivity disorder
>-Tic disorder
>-childhood schizophrenia
>
>Q31. Child adopted. Does not show stranger anxiety and goes off with
>strangers
>
>Q32. F, 37 years has problems at work for several months, she also has
>episodes of hyperactivity & euphoria. These were preceded by episodes of
>sadness &
>inability to cooperate with her colleagues in spite trying hard to do her
>best. What is the diagnosis?
>-Bipolar disorder
>-Dysthymia
>-Cyclothimia
>-Masked depression
>-Factitious disorder
>
>Q33. Anorexia Nervosa A/E
>-Bradycardia
>-hypertension
>-mediastinal air
>-atrophic breast
>-dental crown
>
>Q34. What is Delusion?
>-a persistent belief contradicting
>-a fixed thought
>
>Q35. Among antipsychotics what is correct?
>-Haloperidol-
>-Olanzapine properties
>-Risperidone causes prolactinemia and increases sexual activity
>-clozapine properties
>
>Q36. 17yr old girl comes with depression. She is treated with Sertraline.
>Then she admits abusing amphetamine. What should be done-
>-Stop Setrtraline and give paxel
>-Give written instructions about amphetamine abuse
>-Inform family
>
>Q37. Pt on carbamazepine
>with new onset seizures, 3 attacks in last 2 weeks. Serum level of
>Carbamazepine is normal. What to do?
>-CT scan and EEG
>-Give another agent- phenytoin
>
>Q38. 32 yrs old pt presented with decreased concentration, slowness of
>thinking apathy, socially withdrawn, short term memory loss with difficulty
>in learning new information then developed seizure. What is the diagnosis?
>-AIDS-dementia complex
>-Alzheimer¡¦s disease
>
>Q39. Tardive diskinesia features on haloperidol DOC- clozapin
>
>Q40. Schizophrenia is most commonly associated to
>-monozygotic twins
>-dizygotic twins
>-siblings
>-family history
>
>Q41. A man admitted for operation present with confusion on the 5th post-Op
>day. Which one is a feature of delirium?
>-Depressed mood, preoccupation with suicide.
>-thinks himself the best person and should be shifted to the best ward
>
>-Is upset with shadows and constantly asks what time it is
>-Thinks of buying everybody a jacket
>
>Q42. Case of drug addiction with rhinorrhea, tearing, tachycardia, dilated
>pupils & high BP. Most likely cause is:
>-Cocaine
>-LSD
>-Heroin
>-opiod withdrawl
>-Barbiturates
>
>Q43. Child 9 years old studying in kindergarten is unable to read, write or
>even to color a picture. He becomes happy when he answers simple questions.
>What is your diagnosis?
>-Autism
>-Mental retardation
>-Specific learning disability
>-ADHD
>
>Q44. Paranoid ideations in substance abuse, what is the drug- choices I dnt
>remember
>
>Q45. A child having problem with words, cannot distinguish between turn and
>over. Also causes frequent grammer errors. What is the diagnosis?
>-Language disorder
>
>Q46. A 9yr old child eats glue, pencils etc. teacher
>controlled his behaviour by placing vegetables on his table he started
>eating that, this is an example of?
>-ODD
>-conduct disorder
>-autism
>-ADHD
>
>
>POPULATION HEALTH & ETHICS
>
>
>Q47. Drug compliance can be increased by
>-alcoholism
>-more no of drugs for t/t
>-fear of doctor
>-social problem
>
>Q 48. 59 year old female for PHE. What will you advice
>-self examination of breast
>-Bone scan
>
>Q 49. Which of the following has extra human host- pinworm
>
>Q 50. Question on higher socio-economic condition and effect on health care
>system
>
>Q51. A patient with history of occupational exposure 20 years back now
>presents for 6 months with cough and weight loss. CXR shows fibrosis of
>upper lobes. What is the diagnosis?
>-Tuberculosis
>-mesothelioma
>-rapidly progressive silicosis
>
>Q52. Which of the
>following CA is caused by vinyl chloride?
>-Lung CA
>-Esophagus CA
>-pharyngeal CA
>-Liver CA
>-Bladder CA
>
>Q53. What is the best way to appreciate health care quality during
>pregnancy and delivery in Canada?
>-Neonatal mortality rate
>-Perinatal mortality rate
>-Infant mortality rate
>-Maternal mortality rate
>
>Q54. Maximum radiation exposure takes from?
>-Nuclear fallout
>-natural background
>-nuclear reactor
>-X-ray
>-uranium mine
>
>Q55. In surveillance in worker for radiation hazard best will be
>-Annual PE.
>-Total body radiation count
>-CBC every 6 months
>-Chest X ray yearly
>-eye examination for cataract yearly
>
>Q56. All of the following foods can cause cancer except
>-Monosodium glutamate
>-fat
>-alcohol
>-smoking
>
>Q57. Ethics question- a man suspects to be having probable
>gonorrhea. His wife works in lab. He doesn¡¦t want her to know his
>specimen. What to do.
>-ask him to tell his wife
>-put a code no. on the sample, not his name.
>-Don¡¦t listen to him and straight inform his wife
>-send him to another clinic
>
>Q58. Rehabilitation after a car MVA means:
>- primary prevention
>- secondary prevention
>- tertiary prevention
>
>Q59. Occupational hearing loss is characterized by
>- worst at high frequencies
>-worst at low frequencies
>-progressive even if exposure stopped
>
>Q60. ONE of the following statements is wrong
>-A layer of ozone develops near a photocopying machine in closed area
>-Ice skating rink contains higher concentration of NO2 ??? was this choice
>here? Or was that a separate question?
>
>Q61. Proper disposal of waste- what is the best and most efficient way to
>reduce lead poisoning?
>
>-wash the hands thoroughly every time they eat.
>-use disposable outfit
>-wear masks
>
>Q62.Greatest affect on mortality -accidents
>
>Q63. MC cause of peri-natal mortality - prematurity (checked)
>
>Q64.Regarding Pancreatic cancer, which of the following is not a risk
>factor
>-caffeine
>-pancreatitis
>-alcohol
>
>Q65. Clinical scenario on silicosis. Progression of symptoms for the last 6
>months to 1 year. X Ray shows upper lobe fibrosis. Past history of silica
>exposure 20 years back.
>-mesothelioma
>-pulmonary tuberculosis
>-rapid progression of silicosis of lung
>
>Q66. Old man with CVA requires CPR. Terminally ill. No written will. Family
>wants full support. What to do in case of withdrawing support?
>-apply rules and laws regarding euthanasia
>
>Q67. In periodic health checkup, a 55 yr old has come for a yearly checkup
>and
>has no complaints. What should you advice?
>-check urine glucose
>-check occult blood test
>
>Q68. The most important cause of increased complications of measles in
>developing countries
>-Inadequate immunization
>-Inadequate nutrition
>
>Q69. Young patient with vegetative state, no relatives, and patient is
>suffering from a terminal illness no chance of recovery. According to what
>you decide not to pull off the ventilator? Age, coma
>
>Q70. Death certificate, alcoholic, pleural effusion and died of acute
>respiratory failure, what is the cause of death
>-respiratory failure
>-pleural effusion
>-alcohol
>-cardiac arrest
>
>Q71. Mother goes for checkup. She mentions she is against her adolescent
>daughter¡¦s wishes and sternly told her not to use OCP when her daughter
>wanted to use them. She thinks that will help her grow promiscuous. Hearing
>this
>what should the physician do?
>-Inform the mother that sexual activity in this age group is normal
>-privately give OCPs to the girl without the mother¡¦s knowledge
>-
>
>Q72. Which is the least cause of HIV infection?
>-Homosexuality
>-heterosexuality
>-Prostitution
>-Drug abuse (1999)
>-Blood transfusion (1988)
>
>Q73. You gave a new drug to your patients with dementia. A new effect was
>noted. Those suffering from multi infarct dementia improved while that with
>Alzheimer¡¦s not. What should you do before prescribing this-
>-inform pharmaceutical company about possible indication of use
>-can use it in this new indication and it shows advantages
>-
>
>Q74. In which of the following food does botulism more common?
>-freeze packaging
>-vacuum packaging
>
>Q75. All are transmitted feco-orally EXCEPT:
>-HAV
>-EBV
>-Norwalk
>virus
>-Polio virus
>
>Q76. All are true regarding criteria for organ transplantation except
>-Absence of all spinal reflexes
>-Absent corneal reflex
>-Absent pharyngeal reflex
>
>Q77. Risk assessment can be done by
>-cohort study
>-case control study
>-none of the above
>
>Q78. Diabetic patient with gangrene foot refused amputate. What should the
>doctor do?
>-Force the patient undergo amputation
>-use other means to prevent sepsis
>
>Q79. Question on case control study
>Q80. Prophylaxis of contacts of meningococcus- Rifampicin
>
>Q81..Define health promotion -control over health
>
>Q82. 45 year old male on beta blocker therapy works with wood cutting
>machines. Hand cyanosis occurs with working tools
>-give information about occupational hazard
>
>Q83. Competency of patient. Incompetent when?
>-if don¡¦t know the
>nature of assets
>-spends more than he earns
>-suffers from a mental disease
>
>Q84. Doctors note to employer regarding illness of employee. What should
>physician not write?
>-the diagnosis
>-make recommendations at work place
>-make a note on restriction of activity
>-give an idea of probable duration of illness
>
>Q86. What about smoking prevention? Question incomplete
>
>Q87.What measures have greatly reduced the complications of caustic
>ingestion in children?
>-Cartons are made tamper proof
>-esophagoscopy and early management of scars
>
>
>MEDICINE
>
>Q88. Elderly female on Tamoxifen therapy for advanced CA breast with bone
>mets. Became thirsty and increased urination, disoriented, nausea and
>vomiting, confused and agitated
>-Hypercalcemia
>-brain metastasis
>
>Q89. Mother notices her 11 year old girls walks drooping on
>right side. What is the diagnosis?
>-idiopathic scoliosis
>
>Q90. Case of Peanut anaphylaxis. Child with urticaria and severe attack of
>dyspnea with facial edema. What is your management?
>-Cortisol I.V
>-Epinephrine SC or IM
>-Intubation
>-Antihistamine H1 & H2
>
>Q91. What happens in Organophosphate poisoning -cholinestrase inhibition
>
>Q92.question incomplete Vitamin b12 ---- cbc GIVEN SHOWING PANCYTOPENIA
>
>Q93. M. 42 years with dark skin, palpable liver. Father died of cirrhosis.
>Diagnosis ?
>-Wilson disease -Hemochromatsis
>
>Q94. 50 F years with recent operation presented with heavy wound bleeding,
>she gives you history of massive bleeding when she had a tooth extraction,
>what will you find?
>-Increased PTT + Increased BT
>-Increased PTT + decreased BT
>-Normal PTT + Increased BT (Vwd------ increase BT)
>
>Q95. vWD
>management-DDAVP
>
>Q96. Female 60 years old with constipation for many months, she has not
>passed stools for the last 3 days. On examination the abdomen is distended
>but non tender. What is your management ?
>-Laxative
>-High fiber diet
>-Mineral oil
>-Enema
>
>Q97. Huntington disease inheritance. Paternal grandmother died of it.
>Paternal uncle all had. Father died of accident at 35 yrs of age. What are
>the chances the person will have the disease (Symptoms arising from a
>typical presentation of HD usually do not develop until a person is aged 35
>years or older!)
>-50%
>-100%
>-0%
>-25%
>
>Q98. A couple comes for counseling. Brother of the lady has the disease-
>Achondroplasia. What are the chances the offspring will have the disease
>-25%
>-50%
>-100%
>
>Q99. Which has the best prognosis among skin cancers?
>-Basal cell carcinoma
>
>-malignant melanoma
>-junctional naevus
>
>Q100. Question on old age risk factor on living alone, regular tx visits
>osteoporosis, which factor is the most risk for fractures? question
>incomplete
>
>Q101. Clinical scenario with BP low, CVP low, and PCWP=4. What is the
>management?
>-give ringers lactate
>-give inotrope
>
>Q 102. Multiple black warty lesions on the back with stuck on appearance
>-Seb keratosis
>-melanoma
>
>Q103. Clinical scene with H/O syncope, B.P=110/90. Systolic murmur to
>carotid
>-Aortic stenosis
>-Aortic sclerosis
>-AR
>-MR
>
>Q104. Management of DVT
>-give heparin
>-give warfarin
>-IVC filter
>
>Q105. Diabetic with sudden heaviness of left arm and face(jaw). What needs
>to be done immediately?
>-ECG
>-CT scan
>
>Q106. question on chronic stable angina- heparin
>
>Q107. Case of
>occipital headache- no loss of consciousness and dilatation of right pupil.
>What is the diagnosis?
>-basillar migraine
>-atlanto occipital joint affection
>
>Q108. Portal hypertension with variceal bleeding and hypotension. What to
>do? -vasopressin
>
>Q109. Desensitisation is useful in which of the following?
>-Isolated allergy to cats
>-food allergy
>-asthma
>
>Q 110. Acute asthma not given
>-antibiotic
>-sodium cromolyn
>-theophylline
>-salmeterol
>-steroid
>
>Q 111 Pica,constip, cramps -lead poisoning
>
>Q112. Pyelonephritis causative organism -E.coli
>
>Q113. Lung abcess t/t - cloxacillin
>
>Q114. Features of mycoplasma 20y old with fever with history of non
>productive cough but clinically well .. C-X ray shows bilateral basal
>Infiltration. Drug of Choice?
>-Erythromycin
>
>Q115. Cerebellar signs- nystagmus
>
>
>Q116. Signs of lacunar infarct-
>UM signs- clonus and Babinski
>LM signs- fasciculations
>
>Q117. Non obese patient feels drowsy in the morning. Wife complains of
>snoring at night and waking up many times (features consistent with Sleep
>apnea syndrome)
>-sleep study and pulse oximetry 9polysomnography)
>-ENT consultation
>
>Q118. Rheumatoid arthritis patient develops sudden pain and swelling left
>calf and ankle. Thigh is normal. Cause?
>-DVT
>-Rupture of popliteal cyst
>
>Q119. Which of following is the most important for preventing CVA?
>-Hypertension control
>-smoking cessation
>-lipid lowering agent
>-aspirin
>
>Q120. A group of people returning from Rocky Mountains developed diarrhea.
>Cause
>-Rocky Mountain spotted fever
>-giardiasis
>
>Q121. Dull on percussion -Pleural effusion
>Q122. An alcoholic man presented
>to ER poor controlled diabetes. Urine ketone negative. Blood values- HCO3
>very low. serum osmolaity 307.
>-Lactic acidosis
>-Methanol poisoning
>-ketoacidosis
>-nonketotic hyperosmolar
>
>Q123. Drug of choice for obese diabetic type 2
>-Acarbose
>-metformin
>-troglitazone
>-Glyburide
>
>Q124. A type 1 diabetic on insulin therapy taking both regular insulin and
>PHI
>8am 16:30pm
>Regular 4U 8U
>PHI 24U 10 U
>Hypoglycemia at 15:30pm. What to do?
>-decrease PHI at 8 am
>-Substitute PHI with ultra lente
>-Reduce the dosage of regular insulin
>
>Q125. Which of the following is true regarding Chromoglycate ?
>-Contraindicated with steroids
>-Necessary in acute attack of asthma
>-Prevent binding of IgE with cells
>-Prevents histamin from synthesized cells
>
>Q126. What are the beneficial effects with cromolyn
>-helps
>in reducing the dosage of steroids
>
>Q127. Definitive test for chronic pancreatitis is
>-CT
>-ERCP
>-MRI
>
>Q128. A 74 yr old male with platelet count of 350,000 to 400,000. What to
>do?
>-ASA
>-Warfarin
>
>Q129. After airplane travel complains vertigo, tinnitus, moderate hearing
>loss snhl, is hypertensive, Diagnosis?
>-Miners disease
>-Acoustic neuroma
>-Barotaruma
>-Meniere¡¦s ds
>
>Q130. Nomocytic anemia -not seen improvement wit vit.b12
>
>Q131. Clinical scenario of pseudogout ¡VNSAID
>
>Q132. Osteoarthritis
>-Acetaminophen
>-ASA
>-celecoxib
>
>Q133. Facial edema, increase JVP, plethora inv (consistent with SVC
>syndrome) -CXR(not sure)
>
>Q134. Not seen with Solvents -Pulmonary fibrosis(checked)
>
>Q135. M. 72 years present with one year history of Cough n pulmonary
>infiltrate X Ray shows middle lobe
>infiltration of the lung. Diagnosis?
>-abscess
>-aspiration
>-Bronchiectasis
>-chronic bronchitis
>
>Q136. question on anion gap. Values of sodium, bicarbonate and chloride
>given. Na ¡V (HCO3+Cl)= AG
>-20
>-25
>
>Q137. Old man with chronic bronchitis, known smoker, stays alone. Recent
>aggravation of cough. Having asterixis
>-CO2 narcosis
>-hepatic encephalopathy
>-uremia
>
>
>PEDIATRICS
>
>Q138. Child 6 years old brought by his mother with otitis media then
>hepatosplenomegaly and lymphadenopathy at all sites along with fever. His
>blood investigation showed Hb=85g/l, and low platelets. The family history
>is noncontributory. Diagnosis
>-Acute leukemia
>-Infectious mononucleosis
>-Kawasaki disease
>
>Q139. which of the following is not used in JRA?
>-methotrexate
>-steroids
>-physiotherapy
>-multivitamins
>
>-analgesics
>
>Q140. Mother worried about her child because of history of myopathy in
>family. What Investigation to be done 1st ?
>-CPK
>-Muscle biopsy
>-Nerve biopsy
>-EMG
>
>Q141. 3 yr old child presents with stridor and drooling features of
>Epiglottitis. Management?
>-intubation
>-antibiotics
>-tracheostomy
>-xray
>
>Q142. Newborn with small head, small palpebral fissure, small philtrium &
>small eyes & flattened meat facial area. Diagnosis?
>-Fetal alcohol synd.
>-coccaine
>-intrauterine infection
>
>Q143.child born with petechiae, hearing loss and intracranial
>calcification-
>-congenital viral infection CMV
>
>Q144. A child 3 years has BP 138/95. He has a systolic murmur right 2nd
>space, femoral pulse not palpable. Born premature. Diagnosis?
>-coarctation
>-PDA
>
>Q145. 4 years African boy on
>trimethoprim/sulfamethoxazole for tonsillitis presented with Jaundice ,
>Investigations : Hb. 9.8 gm % , reticulocytes count 8 %. what is the most
>likely diagnosis ?
>-Sickle cell anemia
>-Thalassemia
>-Spherocytosis
>-G6PD deficiency
>
>Q146. I am not sure this was there!!!!
>A 9-year-old boy has been referred to you for evaluation of bedwetting. He
>is dry during the day but wets every night. His physical examination and
>urinalysis are normal. Which one of the following is the most appropriate
>method for managing this child?
>-An alarm system that rings when the bed gets wet and teaches the child to
>respond to
>bladder sensations at night.
>-desmopressin (DDAVP)
>-psychiatric counseling before all
>
>Q147. Child 8-11yrs old with bitemporal hemianopia. Diagnosis
>-Craniopharyngoma
>
>Q148. Asymptomatic girl, 2/6 systolic murmur on pulmonary,
>fixed splitting of S2
>-ASD
>
>Q149. Growth delay. Which one is of less importance?
>-H/O parents growth
>-
>
>Q150. Baby 2 or 4 months of age. Microcytic hypochromic anemia. What is the
>cause?
>-Breast feed only
>-mother did not take adequate iron supplementation during pregnancy.
>-Prematurity
>
>Q151. What is true regarding congenital pyloric stenosis?
>-Commonly present at 3 months
>-Associated with metabolic acidosis due to vomiting
>-Visible peristalsis is seen in abdomen
>
>Q152. A child presented with fever & small white lesion on the mucous
>membrane of the mouth followed by generalized macul0-papular rash. What is
>the management?
>-Give ASA to decrease fever
>-Give gamma Immunoglobulin.
>-Notify the public health unit
>-Isolation of the family member
>-acyclovir
>
>Q153.1 week passing hard stool every 2-3 day
>
>- tell the way to relieve colic.
>-rectal biopsy
>-sweat chloride test
>Q154. 4month old-------- check weight and height charts
>
>Q155. A baby with birth wt.3.5kg now 4 weeks weighs 3.6 kg. Mother worried
>not drinking enough milk. What will you advise?
>-Tell her that nothing to worry, as some kids don¡¦t gain much weight in
>the first month
>-start formula feeding
>-investigate the kid
>
>Q156. Homeless mother with one month child ------ assess the wt change in
>one month
>
>Q157. Baby`s development milestone not correct
>-4 month not rolling
>-no social smile at 2 months
>
>Q158. Baby normal after birth. When feeding started, immediate choking and
>aspiration
>-Esophageal atresia
>
>Q159. Symptoms of meningitis------ Streptococcus pneumonae
>
>Q160. Baby cyanosed after birth no improvement with oxygen. PaO2=27mmHg.
>What is the
>diagnosis?
>-TOGV
>-VSD
>
>Q161. Which one of the following is not indicative of sexual abuse in a
>child?
>-Gonorrhea culture
>-HSV
>-HPV
>-vaginal hematoma
>-vulvar laceration
>
>Q162. Impetigo in child
>-oral TMP
>-oral penicillin
>-cefuroxime
>
>Q163. Baby with PDA
>-continuous murmur
>
>Q164. Case of infectious mono
>
>Q165. Celiac disease ideal diet - rice and corn flour
>
>Q166. All the following neonates are prone to hypoglycemia EXCEPT:
>-IUGR
>-Diabetic mother
>-Normal infant born at 36 wks
>-Hypothermia
>-Non of the above
>
>Q167. What is true about 11 y.o weight>120% of ideal?
>-Hypotension
>-Exercise, increase physical activity (checked from T.N.)
>-High density lipoproteins is increased
>-endocrinologist referral
>-diet reduced to 30% less calories
>
>Q168. A child with
>acute otitis media. Treated with antibiotics. 2 episodes in last 3 months.
>What to do?
>-another course of antibiotics broad spectrum
>-myringostomy with insertion of vent
>-myringoplasty
>
>Q169. A child with pyloric stenosis. What is correct regarding his
>condition?
>- it peaks at 3 months of age
>-visible peristalsis
>-x-ray should be done
>-develop metabolic acidosis
>
>Q170. 3month infant anemia cause - Prematurity
>
>Q171. Hearing deficit in newborn not associated with -delayed speech in
>sibling
>
>Q172. In bleeding from vitamin K deficiency in newborn which investigation
>is helpful?
>-P TIME / INR,
>-APTT
>-Bleeding Time
>
>
>OBS & GYN
>
>Q173. A 19 yr old at 10 week of gestation comes with complaint of
>intractable vomiting for one week.most appropriate investigation will be?
>-beta-hcg
>-serum electrolytes
>
>-Blood Glucose
>-NST
>
>Q174. Which one of these is a/w human papilloma virus?
>-Condyloma acuminate
>-Condyloma lata
>-umbilicated lesion
>
>Q175. Labour pain in 36 weeks primi. P/V reveals long cervix. Appropriate
>management will be
>-give diazepam
>-give morphine
>-give epidural
>-observe and reassure
>
>Q176. Lady after a prolonged labor she delivered a 4 kg baby. She is not
>able to urinate. Diagnosis?
>-Urethral trauma
>-Maternal dehydration
>-Uterine atony
>-Bladder atony
>
>Q177. varicella-----immunise and advice contraception for 3 mnths
>
>Q178. 19 week gestation delivery, cervix open. Product extruded is of
>weight 300gms with normal features. -incomptent cervix
>
>Q179. Pap smear showing atypia. Patient asymptomatic. No clearcut lesion
>visible. What will be the next step?
>-repeat pap smear in 3-6 months
>
>-colposcopy directed biopsy
>
>Q180. When to give antibiotics in CS
>-1 hr before
>-after delivery of baby
>-after separation of cord
>
>Q181. Pap smear collection method -Rotate spatula 360 degrees
>
>Q182. NO flow with estrogen n prog. Challenge -Asherman syndrome
>
>Q183. TSS -cervico vaginal secretion and cloxacillin
>
>Q184. Oligohydrmnios is seen in -RENAL agensis
>
>Q185. What is not recommended screening test in pregnancy?
>- routine urine culture is not necessary during 2nd trimester (?)
>
>Q186. Foul smelling vaginal discharge
>-Candida
>-Bacterial vaginosis
>
>Q187. What`s the most worrisome in 42w gestation?
>-Non reactive NST (?)
>-decreased fetal movement
>-polyhydramnios
>
>Q188. Type1 DM-Gestational DM, drug contraindicated - Clorpropamide
>
>Q189. HRT in 60 year old. Regular menses for 10
>years. Then for 3 months amenorrhea.
>-reassure
>-Do endometrial biopsy
>-increase the progesterone component
>
>Q190. On HRT. Does not know whether menopausal. What to do?
>-Stop HRT and measure FSH and LH
>-give estrogen and progestrerone challenge
>
>Q191. Cervix at the level of vaginal introitus in a 60 year ols.
>Rectocele+cystocele. Treatment?
>-Abdominal hysterectomy
>-Vaginal hysterectomy
>-pelvic sling
>
>Q192. With copper T increased chances of infection in which of the
>following
>-Nullipara
>-promiscuous
>-PID
>
>Q193. Post op pt of hysterectomy. POD 7. CXR showing multiple cavities-
>Aspiration. Treatment
>-metronidazole
>-clindamycin
>
>Q194. Brow presentation, management?
>-Caesarian section
>-vaginal delivery if anterior brow
>
>Q195. 8cm ,Simple ovary cyst in a 58 y.o women. Management?
>
>Q196.
>Mg sulfate to mother, what not checked
>-serum creatinine
>-knee jerk
>-liver enzymes
>-respiratory rate
>
>Q197. Pregnancy of 12 weeks. Uterus at level of umbilicus. Beta HCG 68000.
>No gestyational sac in uterus. What is the management
>-suction curettage
>-hysterectomy
>-hysterotomy
>-extraamniotic instillation
>-methotrexate
>
>Q198. H/O anencephaly folic acid dose
>- 4mg -1mg
>
>Q199. G2P0 Rh immunization, previous abortion
>-serial monthly Ab titres and serial amniocentesis
>-give Rhogam
>
>Q200. Vaginal lubrication on sexual stimulation occurs due to increased
>secretion from
>-Skenes gland
>-Bartholins gland
>-Vaginal gland
>-vaginal transduate
>
>Q201. 18yr F posted for surgery. She was given 8 ml of lidocaine 1% and
>diazepam for the procedure. After surgery collapses, HR=45/min,
>BP=80/60mmHg. What is the
>diagnosis?
>-Vasovagal shock due to hypovolemia
>-diazepam allergy
>-lidocaine toxicity
>
>Q202.Which of the following is normally seen in pregnancy- fetal heart
>sound auscultation at 22 weeks
>
>Q203. Endometrosis diagnosed by
>-Laparoscopy
>-US
>
>Q204. Female with incontinence with hissing sound of tap water and during
>straining while laughing or sneezing
>-Stress incontinence
>-detrusor instability
>
>Q205. Ovarian cyst in pregnancy of 8 weeks of 6 cm in size. What to do?
>-laparotomy
>-observation
>
>Q206. Pruritus and erythematous lesion on vulva with satellite lesions over
>the medial aspect of thigh and inguinal fold. what is the predisposing
>disease ?
>-DM
>-CA. vulva (vulvar intraepithelial neoplasia)
>-Lichen sclerosis
>-pubic lice
>
>Q207. PID A/E
>-pain adenexa
>- no mass? (not sure bcz everysymp.was
>there)
>
>Q208. what about 4 degree perineal tear? Question incomplete
>
>Q209. After vaginal delivery episiotomy wound gaping. How will you manage
>-resuture with absorbable suture
>-parenteral antibiotic and sitz bath and topical application with repair at
>later date
>-topical antibiotic only with sitz bath
>
>Q210. .A lady whose mother had osteoporosis wants prophylaxis for
>osteoporosis what to give?
>-Vitamin D and calcium
>-Exercise
>-Analgesics
>-Vitamins
>-Estrogen
>
>SURGERY
>
>Q211. A pt. with multiple rib fracture is agitated and not allowing doing
>any examination in ER. What is the immediate management?
>-Diazepam
>-analgesic to ribs
>-haloperidol
>-morphine
>-oxygen
>
>Q212. RTA # pelvis, prostate not palpable on DRE. Diagnosis?
>-extraperitoneal urethera rupture
>-bladder rupture
>-pelvic haematoma
>
>
>Q213. Pelvis #. Blood in meatus, cannot pass urine, what to do
>-pass foley catheter
>-suprapubic cystostomy
>
>Q214. Nasal intonation in voice of a 5yr old child due to
>-nasal turbinate hypertrophy
>-Hard palate defect
>-a/w Cervical LN swelling
>-thyroid enlargement
>
>Q215. Man suffered electrocution due to electric pole contact. Patient is
>unconscious and clinging to the electric wire. What to do?
>-Start CPR
>-use special insulating protective gloves and extricate the patient
>-switch off the power supply and then resuscitate
>
>Q216. A patient sustained electrical burn and comes to your clinic. O/E a
>burn area 2cm by 6 cm is noted in the forearm with fingers affected. What
>should be the next appropriate step?
>-Do ECG and if found normal then discharge and advise pt for followup
>-Do cardiac enzyme test
>-Admit the patient
>and monitor ECG for 24 hrs
>-Admit the patient and do ECG 3 times
>-ECG must be monitored for three days
>
>Q217. Pt. With frost bit, best treatment is:
>-Put hands in warm water 38-40 degrees for 30 min.
>-IV antibiotics
>-Escharectomy
>-Debridment
>-fasciatomy
>
>Q218. Post operative case of CA breast with modified mastectomy done and pt
>presently on Tamoxifen therapy. Which screening will be ideal for screening
>recurrence?
>-mammography
>-Chest X-ray
>-Bone scan
>
>Q219. Effects of Vasectomy (question incomplete)
>- Impossible to reverse fertility after 2 years
>-No affect on BP
>
>Q220. Fitula-in ¡Vano due to
>-Ischi-rectal Abcess
>-anal fissure
>
>Q221. Pt. bleeding during defecation painful not allow exam
>-Thrombosed external piles
>-anal fissure
>
>Q222. After # in forearm manipulation and plaster cast
>done. Patient develops pain on passive extension. Diagnosis
>-Compartment syndrome
>
>Q223. What is the most appropriate measure in clostridial prevention in
>wounds?
>-Radical debridement of wound
>-antibiotic
>-give anti serum
>-oxygen
>
>Q224. Ankle joint injury with laceration, no distal pulsations on
>Examination Pain Management?
>-Debride
>-Manipulate and feel for pulse
>-apply splint and immobilize
>
>Q225. A case of large cervical LN 3cm„e3cm of rubbery consistency in the
>lateral aspect of neck. What will be your next step?
>-Abdominal US
>-Excisional biopsy
>-FNAC
>
>Q226. 50y haematuria,mass in kidney with inc. blood supply-RCC
>
>Q227. What is the condition most commonly mistaken for Appendicitis in
>children
>-mesenteric lymphadenitis
>-Meckels diverticulum
>
>Q228. most common indication for surgery in
>stone---- severe ureter colic (not sure) urosepsis?
>
>Q229. Flank pain. In IVU, calyses are seen blunted. Dye excretion is
>delayed in the affected kidney. 2-3cm stone is seen in the pelvis in the
>lower 1/3rd of the ureter. What to do?
>-analgesics and hydration
>-lithotripsy
>-percutaneous stone removal
>-Remove the stone by retrograde cystoscopy
>
>Q230. 65 years obese Pt. complains of repeated attacks of strong & sudden
>mid abdominal pain radiating to Lt. flank with pallor and diaphoresis with
>asymptomatic perios inbetween. what is your diagnosis ?
>-Acute pancreatitis
>-Cholecystitis
>-Ureteral stone
>-Mesenteric Ischemia
>-Rupture of AAA
>
>Q231. 74y old underwent TURP. Specimen shows low grade carcinoma in 5% of
>the specimen
>-observation
>-Radical protratectomy
>-hormone therapy
>-radiation
>(checked T.N. old age if T1-
>then observe)
>
>Q232. Prostate a1 adenocar.------ RT+PROSTECTOMY
>
>Q233. H/O prostate cancer showing nodule----- USG guided biopsy
>
>Q234. Man 56 years. Father died of prostate ca. o/e a small nodule right
>lobe. PSA= 2.2. What advice
>-follow up in 6 months DRE and PSA
>-follow up in 3 months PSA
>-us guided Biopsy now
>
>Q235. Old lady with ankle edema at the right medial malleulus, with
>superficial ulcer & surrounding scar. Diagnosis. ?
>-Arterial insufficiency
>-perforator incompetence
>
>Q236. Stasis ulcer causing pigmentary changes. What is the treatment?
>-pnuematic stocking
>- stripping saphenous vein
>
>Q237. Pneumothrax-------------Chest tube
>
>Q238. In Femoral hernia what is common?
>-gut obstruction
>-medial and above the inguinal ligament
>
>Q239. Welding burn management 2nd degree in the upper limb,
>what do u do ?
>-Debrid & skin graft.
>-Debrid puncture blisters & bandage with topic cream
>-Leave the wound open & systemic antibiotic
>
>Q240. Breast ca operated, used ASA, bleeding, what to do
>-platelet transfusion
>-give desmopressin
>
>Q241. A 70-yr. old man complaining of pain in his mouth. You examine him
>and he has a flat ulcer on the gum of the lower jaw, near the molar teeth
>area. He tills you that his denture has recently become loose and ill
>fitting. What is the diagnosis?
>-traumatic ulcer
>-ulcerating carcinoma of the buccal mucosa
>
>Q242. Post-op pt continued to bleed despite 10 units of stored blood
>transfusion.
>What is the cause of the continued bleeding?
>-Dilutional thrombocytopenia
>-Hypercalcemia
>-Hyperkalemia
>
>Q243. Young man came with history of automobile accident, multiple anterior
>chest
>FRACTURE and hoarseness, chest x-ray shows widened mediastinum, what¡¦s
>your immediate management?
>-Pericardiocentesis
>-Support the anterior chest fracture
>-Intubation
>-Aortic angiogram
>
>Q244. 35yr old female suffering from LLQ pain with non bloody stool.
>Tenderness present over the area. No fever. Normal rectal examination.
>Diverticulum seen in a barium enema. What to do?
>-give antibiotic prophylaxis for 2 weeks
>-try high fiber diet
>-do colonoscopy
>
>Q245. Hearing loss in old age. O/E AC>BC more in left. Weber lateralizes to
>right. What is the most appropriate?
>-diagnosis of sensorineural deafness
>-conduction deafness
>-recommend using hearing aid
>-recommend audiometry test to be done
>
>Q246. Irregular and constricted pupil with reduced reaction to light in
>-acute glaucoma
>-acute iritis
>
>Q247. 70yrs M with
>total hip replacement complains of distention and obstipation in the post
>operative period. Caecal diameter found 10cm in x ray. What is your
>diagnosis?
>-Oglive syndrome
>-Caecal volvulus
>-sigmoid volvulus
>
>Q248. Man with numbness in the back of the leg and unable to dorsiflex the
>foot along with severe backache and stiffness & no history of urine
>incontinence.
>-OSTEOMYELITIS T12- L1
>-peroneal nerve injury
>-cauda equina syndrome.
>
>Q249. Thyroid swelling in a hypothyroid patient. What is to be done?
>-give L-thyroxine
>-Thyroid scan
>-FNAC
>
>Q250. Swelling of rt upper extremity of a female with no previous history
>due to
>-venous embolism
>-lymphedema
>Q251. Lesion on lip - Biopsy
>
>Q252. Renal injury bleeding- pyelogram
>
>Q253.
>Pre op most worrying -H/o MI -age over 70
>
>Q254. A case showing symptom and signs of complicated appendicitis
>
>Pictures
>
>Q255. Basal Body temperature chart of a woman (anovulatory cycle) -short
>luteal phase
>
>Q256. Vaginal lesions- diagnosis
>-vaginal herpetic lesion
>-moniliasis
>
>Q257. Picture of slide showing T. vaginalis. What is the treatment
>-oral Metronidazole
>-ceftriaxone
>-doxicycline
>-topical application.
>
>Q258. Picture of a child suffering from constipation showing large
>protruding tongue. Diagnosis?
>-hypothyroidism.
>-Downs syndrome
>
>Q256. Picture of SKIN lesions involving nails and hand which are suggestive
>of PSORIASIS . What is common?
>-DIP involvement
>
>-Sacroilitis
>
>Q260. Picture of a child having scaling yellowish over scalp and dry face.
>-Seborrhic Dermatitis
>-Atopic dermatitis
>-PSORIASIS
>
>Q261. ECG strip showing variability of R-R interval and RsR¡¦ pattern in
>lateral leads and lead I
>-Atrial fibrillation with LBBB
>
>Q262. Picture of a short stature girl with no sexual development, no pubic
>hair, no breast development (Turner syndrome). What should be done?
>-karyotype
>-FSH and LH
>-TSH & GH
>
>Q263. Fetal heart rate monitoring strip (showing late decelerations?) 32
>weeks with uterine contraction. What should be done?
>-Do caesarian section
>-wait and augment
>-give tocolytics
>-give steroids.
>
>Q264. A girl with a SKIN
>lesion with irregular border and variegated appearance. Recently been
>bitten by dog in that area. Diagnosis
>-naevi
>-malignant melanoma
>
>
>Reward your sense of adventure with MSN World Tour
>
>Test your trivia skills! Play MSN World Tour today!
>

_________________________________________________________________

MCCQE CANADIAN BOARD EXAM PAPERS DOWNLOAD FREE





MCCQE Board papers for practice

Solve as much as you can to get the best out of it.
click on the following attachments






Attachments:






==========================
Sample Questions





Attachments:





NOVEMBER 2003

=====================================






Attachments:




SAMPLE PAPERS

===========================================






Attachments:





===============================================






Attachments:




===================================================






5.2.11

IMPORTANT MCQ QBANK FOR FCPS, MRCP1, MCCQE

Pemphigus vulgaris.
Actinic keratosis.
Paget`s disease of bone.
2.A 40 y old man complains of pain in the Rt shoulder, passive movements are normal, active adduction is limited at 30-120 degree. the most likely diagnosis is:
a.Bicipital tendonitis.
b.Rotator cuff problem
c.Rt shoulder dislocation
d.Bursitis
Q1.
A 6 years old child presenting with nocturnal enuresis, what is the FIRST step in management?
A. wet alarm
B. desmopressin
C. cystourethrogram
D. imipramine (not sure if it was there)

Q2.
A 13 yrs old obese boy with limp and hip pain with limited internal rotation and abduction, what is dx?
A. leg calve perthes
B. slipped capital femoral epiphysis
C. osgood schlater disease
D.DDH
Q3.
Jehova witness pt had active card saying dont use blood for him, he is unconscious, but wife says administer blood, what will u do?
A. accept wife request
B. consult ethic committee
C. obtain court order
D. only resuscitate and accept whatever outcome may be.

Q4.
25yrs old woman with weakness, hyper reflexia, and horizontal nystagmus, what is Dx.

A. MS
B. GBS
C. ALS
there were also qs on
alcoholic cardiomyopathy
diverticulitis
acute hepatitis
cholangitis
ascending cholangitis
schizophrenia.
A YOUNG WOMAN WITH PAIN AND NUMBNESS IN HAND AND TYPE WRITER BY PROFFESSION (I THINK) , WHAT IS THE TREATMENT?
4 OPTIONS BUT THE ANSWER WA
*CARPAL TUNNEL RELEASE
a 17 yrs old pt with anorexia nervosa presents with syncope, what investigation will u perform?
i chose,
*serum electrolytes
pt with diverticulosis, best investigation.
colonoscopy/sigmoidoscopy
pt with lesion on forehead with pearly borders.
it was
basal cell ca
child with burns best analgesia
*iv morphine

picture of malignant melanoma

got 2 questions on schizophrenia.
child with redness, swelling, proptosis and reduced movements of rt eye, Diagnosis.
*orbital cellulitis

periorbital cellulitis was also in options so know the defference between these two.
1-You are seeing a 9 month completly breast fed baby for follow up, the mother complains that he can not sit alone without support, what is the most likely diagnosis:
a. Hypothyroidism
b. vitamin D deficiency.
c. hyperparathyroidism
d. Normal for his age

2.A 40 y old man complains of pain in the Rt shoulder, passive movements are normal, active adduction is limited at 30-120 degree. the most likely diagnosis is:
a.Bicipital tendonitis.
b.Rotator cuff problem
c.Rt shoulder dislocation
d.Bursitis

3.I n Paget disease:
a.Serum Ca low, Phosphorous low, Alkaline phosphatase high
b.Serum Ca Normal, Phosphorous Normal, Alkaline phosphatase high.
c.serum Ca low, Phosphorous high, Alkaline phosphatase normal
d.serum Ca high, Phosphorous high, Alkaline phosphatase high

4.A 55 y old male patient, c/o headache, low back pain, and Rt hip pain, blood workup are normal excepy for high alkaline phosphatas, X ray of skull showed Platybasia, X ray of the hip showed cortical thickening, and sclerotic bony changes over the hip, what is your diagnosis;
a.Paget disease of the bone.
b.Multiple myeloma
c.Ankylosing spondylitis
d.Perth’s disease

5.which one is the most essential content of the informed Consent:
a.The nature of the treatment or investigation, and its expected effect.
b.The signature of the patient
c.The alternative treatment, and its risk
d.The consequences of declining treatment

6.According to the researches in Canada, which of the followings disease increased in the last 20 years:
a.Mental retardation
b.Obsessive compulsive neurosis
c.Schizophrenia
d.depression
e.Anxiety disorders

7.a 27 year old male patient, he went for camping on the beach with his girlfriend for 2weeks, one week later he see you for a skin rash over his neck and chest (see the picture, ?Tinea versicolor), what is your management:
a.Hydrocortisone 1% ointment
b.Ketoconazole (Nizoral) ointment and shampooing.
c.Erythromycin 250 mg tab Qid
d.Hydrocortison 2% cream
9.You are called to see a newborn 30 minute after normal vaginal delivery of diabetic mother, his Apgar scores was 9 at 5 minutes, he is dyspnic, RR 60 per minute, O2 sat 83% on 100% oxygen, chest x ray showed air bronchogram with reticulnodular changes on both lung fields. What is your diagnosis
a.Resp. distress syndrome.
b.Transient tachypnia of the newborn
c.Choanal atresia
d.Pneumothorax

10.A newborn baby delivered 1 hour ago, dyspnic, RR 70/minute, Chest is hyperinflated, Heart sounds are not heared, brething sounds normal on Rt side and no breathing sounds on the Lt side, What is your likely diagnosis:
a.Resp. distress syndrome.
b.Transient tachypnia of the newborn
c.Choanal atresia
d.Tension Pneumothorax.

11.you are in emergency department, you see patient with electric shock, he is alert, his pulse is regular, ECG normal, the inlet was on the left hand, and the exit on the RT shoulder, what is your management;
a.Discharge him home, and if he has chest pain to com to the hospital
b.Do cardiac enzymes, if normal discharge him
c.Admit him in intensive care.
d.Call cardiologist on call

12.what is the most reliable sign in Tension pneumothorax:
a.Deviation of the trachea to the opposite side.
b.Paradoxical pulse
c.Flial chest
d.Abdominal breathing

13.An old man with Osteoarthritis, he has limitation of flexion of Rt knee to 30 degree, what is the appropriate treatment;
a.Acetaminophen plus codeine
b.Refer for knee replacement
c.Physiotherapy
d.Local intra-articular steroid injection

14.A 15 year old male patient, known to have Marfan syndrome, after exercise he has sudden deterioration of his vision, what is the most likely cause for his vision problem:
a.Retinal detachment
b.Lense dislocation.
c.Retnal artery occlusion
d.Viterous hemorrhage
15.A 23 year old female patient presented with weakness of right side of the body with exaggerated reflexes, ataxia, and haziness of the optic nerve on the Lt side, what is the most likely diagnosis:
a.Motor neuron disease
b.Multiple sclerosis.
c.Ischemic stroke
d.Brain tumour

16.A74 year old male patient presented with headache, see photo of his right side of the head, which of the following is true in this patient:


a.Jaw claudication is a common presentation
b.If untreated it may cause blindness.
c.Headache is usually in the right side
d.Treatment is mainly nonsteroidal anti inflammatory drugs

17.A 65 year old man with ca lung came with polyuria, polydipsia and dehydration what is the diagnosis:
a.hypercalcemia
b.Brain metastesis
c.Bone metastesis
d.SIADH

18.A 15 year old boy with ALL on maintenance chemotherapy, which of the following is right:
a.you allow him to go to school
b.you put him in isolation room at home
c.you see him weekly if absolute neutrophil count less than 500

19.You are on call in the medical ward, you are called to see an old man with multiple strokes on mech. ventilation, , he has fever, when you reviewed his record, you discovered that his physician is treating him fully including mech. Ventilation, against the patient’s well, what is your action:
a.Treat the patient (septic workup, antibiotic)
b.Discuss the case with his attendant physician
c.Inform the chief of staff that the patient is not properly managed
d.Do not treat the patient and disconnect the ventilator

20.In Procedentia:
a.You see the uterus protruded into vagina when the patient is coughing
b.You see the uterus protruded into vagina when you do pelvic ultrasound
c.You see the uterus protruded into vagina without coughing or straining
d.You see the uterus protruded into vagina when you press on the lower abdominal wall

21.A 34 year lady with connective tissue disease, she was started on daily prednisolon 40 mg what is the best treatment to prevent osteoporosis:
a.Ostrogen therapy
b.Daily tablet of calcium and vit D.
c.Weight reduction and excercise
d.Calcitonin

22.You are a family physician of one patient who came to you for immunotherapy injection, you are busy, and you ask your clinic secretary to give him the injection. The secretary can give him the injection after:
a.You fill the syringe and prepare the injection
b.She should have an official course how to inject
c.You should have a consent that the patient agree not to be injected by the doctor
d.The secretary should be part of the health system

23.A16 y old girl presented with Primary amenorrhea, physical exam is normal, on pelvic exam, you found bulging mass, pinl in color, what is your next step:
a.transvaginal ultrasound
b.Beta hCG
c.Ct scan of pelvis
d.Estrogen replacement

24.A35 year old patient presented with abdominal pain and distension, and he looks sick, abdomen X ray (megacolon)

What is the most likely cause for this pain?
a.Diverticlitis
b.Intestinal obstruction
c.Fecal impaction
d.Ulcerative colitis
27.An 18-year-old woman previously in good health, she came to emergency room 6 times within one month for lightheadedness, headaches and nausea. She appears anxious and is tremulous, sweating and breathing heavily with tingling sensation around her mouth and in her fingers. Which one of the following would be your proper management for this patient?
a.Ask her to breathe into a paper bag.
b.Give Lorazepam tablets PRN
c.Give Busperone tablets PRN.
d.Fluoxetine 20 mg Bid.

28.A 56 year old female on Lithium therapy presents to your office complaining of feeling tired and weight gain occurring for the first time. She is psychomotor slowed and she looks tired. Physical finds her deep tendon reflexes to have a slow relaxation phase, but is otherwise normal. The next thing you should do is?
a.Refer her for psychotherapy
b.Check her TSH.
c.Check liver enzymes

29.An old man on spironolactone, what is the side effect expected in this patient
a.Hyokalemia
b.Hypocalcemia
c.Gynecomastia
d.Hypercalcemia


30.A 16 year old girl presented with angiodema, her mother has similar episodes, what is the most usful test to diagnose the cause:
a.Allergy skin tests for food
b.Serum IgE
c.Serum complement
d.Serum C1 esterase inhibitor (C1-INH) level. (hereditary angioedma)

31.Diabetic patient, not controlled on oral tablets, you start him on insulin, what is the most likely side effect;
a.Lipodystrophy
b.Weight gain
c.Blurring of vision
d.Polyneuropathy

32.In Klinfilter syndrome all of the following are present except:
a.Dysmorphic features
b.Mental retardation
c.Behavioural problem.

33.A 64 year old male patient, diagnosed to have Prostate Ca stage A1, what is your management:

a.Radical prostatectomy
b.Radical prostatectomy with radiotherapy
c.Hormonal therapy
d.No treatment at this stage


34.A n old man with pigmented skin lesion diagnosd as malignant melanoma, what is your next step in management:
a.Surgical excision with 2 cm safety margin
b.Surgical excision with 1cm safety margin
c.Cobination therapy
d.Ct scan chest and abdomen for staging

35.Patient on Lipostat, developed URTI , received Erythromycin for one week, C/O muscle cramps in the lower limbs, what is the test most likely confirm the diagnosis?
a.muscle biopsy
b.EMG
c.Creatinin phosphokinase
d.MRI

36.A 17 y old boy, has uncontrolled asthma, allergy skin tests showed that he is allergic to housedust mites, which of the following can reduce the exposure to dust mites:
a.keep upholstered furniture in the room
b.Wash all bedding weekly in warm water (30-40C) to reduce the dust mite population
c.Encase your mattress, box springs and pillows in unwashable encasings
d.Keep pets out of the house, and particularly, out of the bedroom
e.Install a high-performance electrostatic filter in your central air conditioning and heating system to remove more than 90 percent of household dust and dust mites.

37.Pt. found unconscious on the floor behind the bar, alcoholic, ER agitated semiconscious exam shows laceration on the head, all the limbs can be moved, all the others are normal… what is your action?
a.CT of the head
b.IV. fluid + Observation
c.Discharge

38.pt with bilateral flank masses what is the best way to confirm bilateral obstruction of the kidneys:
a.Ultrasound of kidneys
b.IVU
c.Ascending urethrogram
d.Retrograde pyelogram
e.Radioactive renogram

39.Child with abdominal pain attacks for the last few hours, drawing his legs up during the attack, What is the next step in management:
a.Ba enema
b.CT scan abdomen
c.Hydrostatic enema
d.Ultrasound abdomen

40.All of the following are associated with sleep apnea syndrome except:
a.Hypertension
b.Diabetes mellitus
c.Obesity
d.Hypothyroidism
41. 52 yr male pt scheduled for colorectal surgery in 2 months.currently good medical condition.past h/o MI 3months ago.you advise the pt

a-it is alright to proceed with surgery
b-surg not advised within 6 month of an MI
c-surg can done successfully if he takes his regular meds
d-close monitoring during surg ll certainly avoid all copmlecations
41.Which of is the following thyroid tumors has the best prognosis:
a.Papillary.
b.Follicular
c.Medullary

42.A baby of inter-cousin marriage, what is the most likely condition you may see in this baby:
a.Malrotation of the intestine
b.Pyloric stenosis
c.Annular pancreas
d.Oeophageal atresia

43.Rupyure of extensor pollicis longus is likely to occur in:
a.Duputryn contracture
b.Carpal tunnel syndrome
c.Rheumatoid arthritis
d.Lunate fracture
e.Scleroderma

44.2 senarios of Mallory-weiss tear, what is the best tool for diagnosis
a.Upper GI endoscopy
bBa swallow
c.CT chest
d.Chest X ray

45.A patient presents with depressed mood low self-esteem, sleep problems low appetite,
Fatigue and hopelessness for 3 years .He has maintained his work and relations –ships.
a.Major depressive disorder –chronic.
b.Dystimic disorder
c.Panic disorder
d.Bipolar 2disorder

46.The best treatment for a patient with major depressive disorder with psychotic features is?
a.Electroconvulsive therapy
b.Fluoxetine
c.Haloperidol
d.Imipramin
e.Phenelzine

47.A young women presents to the ER who is violent and psychotic. She displays vertical
and horizontal nystagmus. Most likely diagnosis is
a.Schizophrenia
b.Alcohol intoxication
c.PCP intoxication
d.Cocaine
e.LSD

48.OCP prevents
a. endometrial carcinoma
b. cervix
c. breast cancer

49.Pregnant woman 36 weeks. We can see foot out the cervix .Management;
-forceps
-c/s
-delivery

50.Breast feeding women with DVT .Treatments?
- warfarin
- IV heparin
- Antistreptokinase
- bandage

51. The most accurate diagnosis of Down Syndrome
- amniocentesis
- CVS
- U/S

52.4years African boy on Septra for tonsillitis presents with jaundice. Investigations HB
9.8 % reticulocytes count 8%, Hb electrophoreses. Diagnosis
a. Thalassmia
b GGPDdeficiena
c .sphrenayrosis
d. sickle cell anemia

53.Male 50 years old with prostatic cancer with bone metastasis. Treatment


54.Female vagina bleeding. Pregnancy test. Empty uterus test adrenal mass
- ectopic pregnancy.


55. Which of the following descriptions of diagnostic test is influenced by prevalence.
- specificity
- sensitivity
- accuracy
- PPV
- reliability



56..Primary prevention are except ;
- immunization
- Health related program
- pap smear
- mammography

57.Child RTA with urethral bleeding management
- retrograde urethrogram
1. a lady one year after delivery present with amenorrhea even though wshe chose not to breast feed her baby. she had PPH and curetting done during delivery you gave her estrogen-progesteron but no withrawl bleeding what is your diagnosis?

a) sheejan syndrome
b) asherman syndrome

2. which of the following is most likely to be ectopic pragnancy?

a) 7 week amenorrhea with vaginal spotting
b) 7 week amenorrhea with IUD
c) 12 week amenorrhea with right lower quadrant pain
d) 5 week amenorrhea with menstueal like cramps
Which one of following is true regarding Alcoholic hallucinations
a.Hallucinations are usually auditory
b.best treated with benzodiazipines
c.usually occur in chronic alcoholics who goes on particular heavy binge
d.usually a chronic disorder
e.more common than schizophrenia
A picture of face with a suture above the eye brow surrounding areas is red and swollen.Dog bite wound , what shall be done?
a- remove the suture immediately
b-suturing of a dog bite is not recommended
c-oral abx
d-IV abx
previously well,1 month infant presented in ER with fussiness.vitals temp 37.1rectally,RR50/min HR 235/min BP 85/35.otherwise well profused, which investig do ist

a chest xray
b echo
c blood gasses
d ECG
e sr ca
57 y male in CCU for MI .NOW AGITATED,disoriented hallucinating ,which medication help
a morphine
b phenobarbitol
c chlorpromazine
d haloperidol
e lithium
24yrs G7P6 tertiary care hospital ,active labour 37WK P/V bulging memb and cervical dilation of 7cm,no presting part felt in the pelvis what is 1st action
a ARM in op theater
b real time usg on labour floor to determine presenting part
25yrs male presented as odd, maintaining no eye contact.he is having difficulties obtaining a job, as he gets preoccupied with his interest in royality.he does not report any magical thinking.he has along standing H/O resistance to change,hypersensitivity to noises and minimal social contact, diagnosis
a generalized anxiety disorder
b social phobia
cobseccive comp disorder
c schizophrenia
one more question,
in acute pancreatiti what inves gives exact diagnosis
is it CT With IV contrast
some more q
58. Bier block,which is true
a bupivacain is better
b effect last longer even after the procedure,giving adequate postop analgesia
c suitable for all procedures in hand
d always make the emg tray ready before the procedure
59 in the ER young pt treated with procainamide his BP dec to 80/60, you tried another time with procainamide,his bp still decreasing what to do
a benztropine
b saline perfusion
c digitalis
d defebrillation
e intubation

60 72 yr male w hert failure w high BP treated for long time.he was brought to ER for hypotension80/60 HR 110 what to give

a procain
b lidocain
c cardioversion
d digitaline
e frusimide
105. a pregnant lady had routine blood work at 13 weeks – GTT 5.7 ,VDRL negative, HBS Ag negative, Hb- 13. Rubella titer negative. what will you advice
a. start iron now
b. start folic acid now
c. give rubella vaccine now.

106. which of the following is true about huntington’s chorea
a. AR
b. Both psychological and neurological features are required for diagnosis.
c. Either one can be seen alone early In the course of disease.

107.which of the following feature distinguishes maxillary from zygomatic fracture
a. diplopia
b. malocclusion

108. which of the following is true.
Cluster head ache always bilater and develops gradually
Migraine always unilateral
Migraine commonly associated with nausea, vomiting,photophobia,
Common head ache always unilateral

110. OCD patients seek medical attention after 5-10 years of disease onset. Because
a. they think their feeling is normal
b. they are afraid to see doctors.
c. fear of social isolation
d. they feel shame of their obsessions and compulsions

111. a 25 year old man complaints of arthritis, conjunctivitis and urethritis. Which of the
following seen with this
a. mouth ulcers
b. skin rashes

112. 18 month old child does not speak a word. He seems to understand everything his mother
says to him. He waves bye bye when she leaves. Started cruising at 12 months and walking at 15
months. What will be your action
a. developmental assessment
b. reassure and regular follow up.

113. which of the following is not a dysmorphic disorder (Not sure)
a. Williams syndrome
b. prader villi
c. down syndrome
d. angel man syndrome.

114. a man while lifting a heavy weight developed sudden pain in back radiating to leg.
Involvement of L5 root. Which of the following is true?
A. inability to dorsiflex big toe
b. heel walking is not possible
c. toe walking is not possible.

116. 28 year old man , painless scrotal mass , tans illumination negative. What is the most
appropriate investigation\
a. needle aspiration
b. open biopsy
c. USS

118. a child with chronic renal failure, with posterior urethral valve. He is prepared for a
ventilation tube insertion. Nil orally for last 12 hours. Throughout the procedure 70 ml /hour
ringer lactate given.
s.Na. is 150. what is the reason for hypernatremia.
a. decreased concentrating capacity of kidney
b. excess administration of IV fluids.
c. insensible water loss

119 a child with green stick fracture with 10 degree angulation. Treatment.
A. POP cast
b. correct angulation to zero and put cast
c. open reduction
d. splint
Which of the following findings is most consistent with a diagnosis of learning disability?
a. Normal score on Intelligence test
b. Normal score on Connors Rating Scales for rating hyperactivity
c. Abnormal findings on Audiometry
d. Abnormal findings on EEG***
e. Abnormal findings on neurologic examination


CDM:

26 year old male, known schizophrenic brought in to ER. Very angry, and threatening to kill his girlfriend.

What is your management? (upto 3 type in answers)

32 year old woman from Nunavet, who is very ill is flown in by helicopter. She is diaphoretic, hypotensive, diarrhoea+. History is she ate a traditional nunavet smoked fish delicacy sometime earlier.
What will you do:

several options to choose with no limit on how many.

this was very public health type question, with options such as
?report to health ministry
?test stool toxin all relatives
?isolate patient
forget the other options..was a hard question, was very tempting to select everything and had no idea what this woman was suffering from. If anyone else who did the exam remembers please fill in the blanks.

Mother brings 8 year old son who has missed school for 3 weeks due to constant nausea, several episodes of vommiting. All blood tests, and physical exam is normal.
Type in your two top diagnoses.

32 year old lady has been trying to have a baby. Was on contraceptions for several years and stopped 6 months ago. What is your management?

Several options given for the usual investigations for infertility and you are allowed to select as many as you like. Only chose the "reassess patient in 6 months" option as you have to wait at least a year.

Acute MI sounding case description with a gentleman have crushing chest pain and breathlessness...what is your management?

Type in upto 5 things.

Painful knee in a 8 year old child. What is your management. Several options to choose.


5 year old boy from a car accident has a ruptured bleeding spleen. Surgeon on call says if they do not operate soon child will die. Hb is dropping in the meantime. Parents are Jehovah witnesses and do not want blood products.

Cancel operation and let boy die as he will most likely die on the operating table?
Give blood anyways without consent?
Try operation without blood products?

10 year old boy, not good at spelling or reading, but does well in math. Constantly in trouble at school and does not get along with others.

Type in your two diagnoses.

G1P0 Woman at 38 weeks , cervix dilated 2cm having varied but painfull contractions for the last 6 hours.

Type in your diagnoses.

What is your management, choose as many as you can?

?Oxytocin
?Amniotomy
?Watch and wait
?C section
?Intrauterine pressure measurement
?Ultrasound
?Magnesium Sulphate
?Epidural anaesthesia
?Place patient on there side
etc.

Sorry this is all I can remember for now.

I remember these ones because they seemed challenging. There was ALOT of management questions, so make sure you remember how to SPELL the drugs. Not sure if they wanted doses, so if you know the dose write it down as well.

There is plenty of time for the CDM, so dont rush it.

The MCQ is a bit of a blur at the moment for me. Not sure how I did, as the questions seem to be getting easier towards the middle to end of it.
They are not unlike many of the questions you will see in the practice MCC exams. Dont neglect your public health questions and ethics either which is what i think I came down on.
Things to do with which health organisation in canada does what will be usefull to review and know well.

Best of luck to you all. It is a looooong day so sleep well the night before, no last minute heroics.
An 11-year-old girl presents to the Emergency Departmentwith a history of recurrent heavy vaginal bleeding.She is hemodynamically stable. Her hemoglobin is 105 g/L, andher coagulation studies are normal. Which one of the following is your initial management?


Administration of fibrinogen.
Hormone administration.
Blood transfusion.
Dilation and curettage.
Intramuscular iron therapy *********


A 21-year-old man is involved in a motorcycle collision. On arrival in the Emergency Department, he is noted to have no movement of his legs, no sensation in his legs and loss of rectal tone. Which one of the following may be beneficial?


Opioid antagonists
Spinal cord cooling
High dose antihistamines
High dose steroids ********( I guess it can be an option but TN says no proven efficacy )??!!!!!
Revascularization of the spinal cord********* ??



A 5-year-old boy presents with periorbital edema. Which one of the following is the best initial management?

Sinus x-rays.
Trial of oral antihistamine/decongestant.
Referral.
Look for other signs suggestive of allergy.
Urinalysis (urine microscopy).****************


A 10-month-old native Canadian (Aboriginal) is flown to your hospital because of a fever of 38.5°C and a stiff neck. Which one of the following investigations is the best way to differentiate an aseptic meningitis from a tuberculous meningitis when examining the cerebrospinal fluid?


Gram stain.
Total cell count.
Latex agglutination.
Presence of mononuclear leucocytes.*************???
Protein content.


A 16-year-old girl treated with fluoxetine 20 mg per day returns to your office after two weeks complaining that despite taking her medication every day, she is still having problems with her mood, appetite and concentration but is not actively suicidal. Upon further inquiry, there is some improvement in her sleep and energy. She is tolerating the medication. What would be the most appropriate action in addition to continued monitoring?


Switch to venlafaxine, 37.5 mg per day
Continue fluoxetine, 20 mg per day
Add bupropion, 100 mg per day
Increase fluoxetine to 30 mg per day********************
Add risperidone, 0.25 mg per day


A pharmacist reports "double doctoring" of a controlled drug by your patient. Which of the following actions is the appropriate first step? To who you report it??????? *******(YOU DONT REPORT YOU TALK TO YOUR PT FIRST)*********

Management for Baker's cyst? *****( Conservative versus sx)****


A patient suffering from psychosis who has not been compliant with his medication, threatens harm to a coworker. The physician considers the threat to be serious and imminent. Which one of the following is the most appropriate initial action? **********(call th police)********

Compared with control subjects, patients with major depression generally show what change in REM and 4 stage????
milestone question what can a 2 year old do walk down stairs with alternating feet
stand breifly on one leg
lots of ethic with jehovas witness child going for surgery against parents will for transfusion bp droping fast.what do u do
collegeue suspected of drinking doing poor at work what do u d who do u report to
where and when was extra bililling stopped something about quebe doing that in 1984
infant with signs of maternal viral infection it was staight forward dont worry
had no calculation qs
pneumoconiosis is a.disability,b invalid,c handicap
cohort and case controle easy .dont worry
pic of child with chemical ingestion what do u do gastic laverge,esophagoscopy, ipecac, alcalie etc.
good samaritan law, a ethical duty to help,b. no obligation to help c. obligated to help etc
clear case of pyloric stenosis

i found cdm to be tougher cos there r so many options but thank god its just 25 percent of the whole exam
differentials for post triangle of neck 10
treatment of child hood obesity 10
old man with loss of consicousness seixure dizzy, tinnitus wife noticed this give differentials
acute MI treatment 5
outline management of pulm Embol 10
I had an exam second time to day,
MI case what's the diagnose write initial management
Pulmonary Emblisem case whats the diagnosis, management
Case of Pajet disease asked for DD
Compartment Syndrome case, what's thee management
Uterine atony post partum diagnosis whats the management.
Case of status epilapticus diagnosis and management, make sure to write down IV Im or what ever.
5 days infant with jaundice, diagnosis and managment.
Case of praimay pulmonary hypertension in 35 years old lady.
lump in breast of 32 years old lady, make sure not to use mammography.
Sorry so tired I cant remember more.
Very important tip: in compare of before most of the questions from afternoon part they ask to chose as much as possible so make sure don't chose a lot because you lose whole the point so chose even one or two that for sure you think they are right.
Again thanks for all those that spend their time to post the questions after exam.
. i did get the question about the elderly man with pain in his back,differential diagnosis with 10 spaces
.a mother who brings her female adolescent child who is being teased for being fat on the playground.asks you for advice about the childs management
.patient post surgery develops sob and chest pain. choose what your immediate management will be. seemed like PE. then you were to write out the management after initial treatment based on fact that patient was getting worse
.elderly lady well controlled DM comes in for periodic health exam. she is on glyburide, metformin, ramirpil, hydrochlorothiazide and all is well controlled except she complains in the last weeks she has been having dizzy spells where she is nauseous and diaphoretic. her glyburide was increased 3 years ago due to poor control of her DM. give 2 recommendations to patients management at this point. then she tells you her husband was moved to a nursing home due to his dementia and she goes to visit often though it is getting more difficult becasue winter is coming and she is uncomfortable because it seems someone at the home always has a cold. write 3 recommendations for the patient
.eldery lady who presented with a scenario of rectal bleeding. looke dlike diverticulosis. asked for investigations o confirm diagnosis. then management of teh patient after the acture episode has passed.
. case of acute pulmonary edema.middle age lady with history of HTN, DM with all the medication. brough tto the emergency room in th emiddl eof night by her husband due to worsening os SOB.her "bronchitis" became worse in the last 2 weeks. she has bibasal crackles, diffuse wheezes, pitting edema 2+, bp is high, HR high, RR high. state management 5. choose up to 8 or 10 options that could have casued her situation. she is diabetic too

in mcq there were a lot of ethics questions esp with relation to provincial/territorial regulatory body. i remember a question about a guy who fell off teh scaffolding at work and had colles fracture. you treat the guy at your clinic but who do you send his bill to? provincial health insurance, employer, workers compensation board, etc

there was another guy who worked fixing ceilings and had a problem with his shoulder. could not abduct nor internally rotate. write diagnosis and management.
I got mcqs about immunization , milestones,and rash..try to memorize those before you go to the exam.
As for the CDM..I got cases for MI,COPD,PE,Epilepsy,OCD,personality disorder,ortho questions about 4 ,diverticulitis.
Most of my cases were asking me to write the Mx ,no ddx this time..
dissociative amnesia, I was asked two question on that.
diagnosis of chronic sinusitis.
BRCA1 and 2, relation to breast and ovarian cancer.
post operative patient with colon distention to 10 cm, diagnosis

1) A family physician cares for a family consisting of a 45-year old husband, 43-year-old wife and a 12-year-old daughter. The family reports that recently the 77-year-old maternal grandmother who lived with them died after a prolonged respiratory infection. Autopsy subsequently confirms that she had active pulmonary tuberculosis at the time of death. The organism tested sensitive to all anti-tuberculosis drugs. In responding to the grandmother's illness, which of the following is the most appropriate step in management?
A. Obtain leukocyte counts on all family members
B. Obtain sputum cultures for acid fast bacilli
C. Obtain chest computerized tomograms on all members
D. Place protein purified derivative (PPD) test on all members
E. Schedule bronchoscopy lavage for the adults
Explanation:
The correct answer is
D. The immediate step is to screen the family for TB exposure. The most effective manner in which to accomplish this is by placing PPDs on all members and working up those with a positive test. The white cell count may be elevated for a variety of reasons and would not necessarily help in diagnosis or management (choice A).

Sputum cultures will take 6 months to grow and may be too cumbersome to obtain (choice B).

Chest CT scans may show the tuberculosis lesion but a more effective method would be to place the PPD and perhaps then scan those with a positive test (choice C).

A bronchoscopy would be too invasive an option at this point (choice E).



2) A 57-year-old man comes to the emergency department because of excruciating pain in his right big toe. He describes the pain as so severe that it woke him from a deep sleep. He has no chronic medical conditions, does not take any medications, and denies any similar episodes in the past. He admits to a few "drinking binges" over
the past 2 weeks. His temperature is 38.1 C (100.5 F), blood pressure is 130/90 mm Hg, and pulse is 80/min. Examination shows an erythematous, warm, swollen, and exquisitely tender right great toe. The skin overlying the first metatarsophalangeal joint is dark red, tense, and shiny. Synovial fluid analysis reveals negatively birefringent, needle-shaped crystals within polymorphonuclear leukocytes (PMNs). Laboratory studies show:
Serum
Leukocytes........16,000/mm3
Uric acid...........15 mg/dL
Calcium.............9 mg/dL
Which of the following is the most appropriate pharmacotherapy?
A. Allopurinol
B. Ceftriaxone
C. Indomethacin
D. Probenecid
E. Sulfinpyrazone
Explanation:
The correct answer is
C. This patient has the classic presentation of a patient with acute gouty arthritis with the sudden onset of severe pain (typically in the middle of the night), swelling, erythema and warmth of a single joint. Low-grade fever and leukocytosis may be seen. It is more common in men and it is associated with hyperuricemia, usually due to decreased renal excretion of uric acid. Common causes are thiazides and alcohol. Diagnosis is made by examination of joint fluid under polarizing light. Negatively birefringent, needle-shaped crystals within polymorphonuclear leukocytes, hyperuricemia, and acute monoarticular arthritis make the definitive diagnosis of gout. Indomethacin or colchicine is the treatment during an acute attack. Allopurinol, probenecid, and sulfinpyrazone are used for prophylaxis against further attacks.

Allopurinol (choice A) is a xanthine oxidase inhibitor that is used as an antihyperuricemic agent by individuals with recurrent gouty attacks. Common side effects include rash, headache, and gastrointestinal upset.

Ceftriaxone (choice B) is the treatment of acute gonococcal arthritis. It has no role in the treatment of gout.

Probenecid (choice D) is a uricosuric agent that increases the rate of urate excretion. It is used to prevent gouty attacks. It may precipitate nephrolithiasis.

Sulfinpyrazone (choice E) is another uricosuric agent that increases urate excretion. It is used to prevent gouty attacks. It, too, may precipitate nephrolithiasis.

3) A 35-year-old woman arrives on the floor after an uneventful hysteroscopy to evaluate her long history of uterine fibroids. About 30 minutes after her arrival, she begins to complain of nausea and has two episodes of vomiting. The physician administers 0.625 mg of droperidol and 400 mg of acetaminophen by mouth. On follow-up evaluation, the patient's neck is involuntarily flexed to one side. She is alert, oriented, and conversant and has an otherwise normal neurologic examination. Which of the following is the most likely diagnosis?
A. Cerebral vascular accident
B. Conversion disorder
C. Dystonic reaction to droperidol
D. Munchausen syndrome
E. Seizure
Explanation:
The correct answer is
C. The most likely diagnosis is a dystonic reaction to the droperidol. Droperidol causes its antiemetic effect by antagonizing dopaminergic receptors in the vomiting center (central chemoreceptor zone) of the brain. This antidopaminergic action can produce torticollis or other dystonias.

A cerebral vascular accident (choice A) is unlikely given that the patient is alert and oriented, has no detectable language deficit, and has an otherwise nonfocal neurologic examination.

A conversion disorder (choice B) is unlikely since the patient has no prior history of a psychiatric disorder and has a viable medical reason (dystonia from droperidol) for her neuromuscular deficit.

Munchausen syndrome (choice D) is also unlikely since the patient had valid medical reasons for her initial admission and your current visit. We are also not informed of any prior history of hospitalizations or seeking of medical attention without appropriate cause.

A seizure (choice E) is similarly unlikely since the patient has no history of a seizure disorder and is alert, oriented, and conversant.

4) A 50-year-old nurse consults a physician because of a rash above both her ankles. Physical examination demonstrates marked ankle edema with erythema, mild scaling, and brown discoloration of the overlying skin of the distal lower legs. Varicose veins are also noted. Which of the following is the most likely diagnosis?
A. Atopic dermatitis
B. Cellulitis
C. Lichen simplex chronicus
D. Nummular dermatitis
E. Stasis dermatitis
Explanation:
The correct answer is
E. This is stasis dermatitis, which is a persistent inflammation of the skin of the lower legs. The condition is often related to varicose veins, although it has been postulated that the true cause may instead be perivascular fibrin deposition and abnormal small vessel vasoconstrictive reflexes. The presentation illustrated is typical. Most patients are relatively asymptomatic and may not seek medical attention until the edema becomes severe or the lesions become complicated by secondary bacterial infection or ulceration. It is important to increase the venous return to the heart by elevating the ankles while resting and use of properly fitted support hose. Local topical tap water compresses can be helpful. Purulent lesions can be treated with hydrocolloid dressings. Ulcers are treated with compresses and bland dressings, such as zinc oxide paste.

Atopic dermatitis (choice A) typically involves the antecubital and popliteal fossas, eyelids, neck, and wrists.

Cellulitis (choice B) is a bacterial infection of the subcutaneous tissues, and causes local erythema, tenderness, and often lymphangitis.

Lichen simplex chronicus (choice C) is a skin rash caused by chronic scratching characterized by dry, scaling, well-demarcated, hyperpigmented plaques.

Nummular dermatitis (choice D) causes widespread coin-shaped, crusted skin lesions.

5) An AIDS patient develops symptoms suggestive of a severe, persistent pneumonia with cough, fever, chills, chest pain, weakness, and weight loss. The patient does not respond to penicillin therapy, but goes on to develop very severe headaches. The presence of focal neurologic abnormalities leads the clinician to order a CT scan of the head. This demonstrates several metastatic brain abscesses. Biopsy of one of these lesions demonstrates beaded, branching, filamentous gram-positive bacteria that are weakly acid fast. Which of the following is the most likely causative organism?
A. Actinomyces
B. Aspergillus
C. Burkholderia
D. Francisella
E. Nocardia
Explanation:
The correct answer is
E.Nocardia asteroides is an aerobic soil saprophyte that can cause acute or chronic infectious disease often characterized by granulomatous-suppurative lesions that may become widely disseminated. Many, but not all, patients have underlying causes for immunodeficiency, including advanced age, lymphoreticular malignancies, organ transplantation, high dose corticosteroid therapy, or (increasingly commonly) AIDS. Disseminated nocardiosis usually starts as a pulmonary infection that can resemble either a severe pneumonia or tuberculosis. Once dissemination occurs, metastatic brain abscesses are particularly common, occurring in as many as 1/3 of patients with nocardiosis. Nocardiosis is treated with sulfa drugs, such as sulfadiazine or trimethoprim-sulfamethoxazole, for periods of months.

Actinomyces (choice A) is very similar to Nocardia, but is not acid-fast.

Aspergillus(choice B) is a fungus.

Burkholderia(choice C)pseudomallei is a gram-negative bacillus that causes melioidosis, which is characterized by lung involvement or disseminated infection.

Francisella(choice D)tularensis causes tularemia, which is usually acquired by contact with infected wild rabbits.



6) A 28-year-old patient with end-stage renal disease (ESRD) on continuous ambulatory peritoneal dialysis (CAPD) for two months presents with fever, abdominal pain and cloudy dialysis fluid. There is no diarrhea or vomiting and the pain has been present for about 12 hours. The patient has ESRD secondary to chronic glomerulonephritis, there is no history of diabetes, urinary infections or antibiotic use. Examination reveals a temperature of 38.9 C (102 F), and blood pressure of 110/70 mm Hg. The throat is clear, as are the lungs. Cardiac examination reveals a grade 2/6 systolic murmur. Abdominal examination reveals decreased bowel sounds with diffuse tenderness. There is mild rebound. There is no edema or skin rash. A complete blood count shows a leukocyte count of 14,200/mm3, hemoglobin is 12.5 g/dL. Peritoneal fluid is cloudy with 1,000 white blood cells, 85% of which are polymorphonuclear leukocytes. Gram's stain of fluid is negative. Cultures of blood and peritoneal dialysis fluid are taken. Which of the following is the most appropriate initial step in management?
A. Fluconazole
B. Immediate removal of dialysis catheter.
C. Intravenous vancomycin
D. Intravenous gentamicin
E. Oral ciprofloxacin
Explanation:
The correct answer is
C. Peritonitis in a patient on CAPD is usually due to gram-positive pathogens such as Staphylococcus aureus or epidermidis. It is usually characterized by abdominal pain and over 100 white blood cells (typically polymorphonuclear leukocytes) in a sample of peritoneal dialysis fluid. Intravenous vancomycin would be a reasonable treatment to cover gram-positive pathogens.

Fluconazole (choice A) would be indicated for a fungal infection. Fungal peritonitis is not usually seen until patients have been treated with multiple antibiotics or are further immunosuppressed.

Immediate removal of the dialysis catheter (choice B) is usually not needed unless the patient has a peritonitis that has not improved with a trial of antibiotics.

Intravenous gentamicin (choice D) has good gram-negative coverage but would not be an ideal drug to cover Staphylococcus.

Ciprofloxacin (choice E) would be a very broad spectrum antibiotic that would not be a first choice as a single antibiotic to treat staphylococcal peritonitis. Further, the oral route may not be adequate as patients with peritonitis may have nausea and vomiting.

7) A 54-year-old man presents for a periodic health examination. His family history is significant for his mother who died of a cerebrovascular accident at age 72, his father who died of a myocardial infarction at age 68, and a brother who developed sigmoid cancer at age 60. The patient is on no medications except for aspirin, 81 mg daily. His physical examination is unremarkable. The patient asks for a recommendation regarding current cancer screening. Which of the following is the most appropriate screening test for this patient?
A. Annual digital rectal examination and fecal occult blood testing
B. Flexible sigmoidoscopy
C. Flexible sigmoidoscopy and barium enema
D. Colonoscopy
E. Genetic testing for the p53 gene
Explanation:
The correct answer is
D. Any patient with a first-degree relative who has developed an adenoma or colorectal cancer should undergo colonoscopy for screening at age 50, or 10 years before the relative developed the adenoma or carcinoma, whichever comes first. This patient has a brother who has a colon cancer at age 60; therefore, a full colonoscopy is warranted. Although there are various opinions regarding appropriate screening in the "average risk individual," there is a consensus that full colonoscopy is required in patients who have an increased risk, e.g., first-degree relative with a positive history.

Annual digital rectal examination and fecal occult blood testing (choice A) are no longer considered a reliable method of screening for colon cancer, since a shift in the demographics of colon cancer has lead to more than half being identified in the first half of the colon. Digital rectal examination also often fails to identify premalignant colonic polyps.

Flexible sigmoidoscopy (choice B) is a good initial screening technique for patients older than 50 with no specific known risk factors. If polyps are identified, they can be biopsied, their type established, and subsequent complete colonoscopy performed if adenomas were identified microscopically.

Flexible sigmoidoscopy and barium enema (choice C) offers an alternative way of screening the entire colon in patients in whom a complete colonoscopy cannot be performed.

Genetic testing for the p53 gene (choice E) is not currently used for colon cancer screening.

8) An AIDS patient under treatment with a nucleoside analog and a protease inhibitor comes to medical attention with complaints of leg weakness and incontinence. His vital signs are within normal limits. Physical examination reveals reduced strength in the lower extremities with accompanying mild spasticity. There is also diminished sensation in the feet and legs bilaterally. Lumbar puncture shows:
Opening pressure.....100 mm H20
Cell count................5 lymphocytes/mm3
Glucose...................48 mg/dL
Proteins, total..........33 mg/dL
Gamma globulin.......8% total protein
Additional laboratory investigations show normal hematologic parameters, vitamin B12 within normal values, and negative serology for syphilis. MRI of the head fails to reveal any focal abnormality. Which of the following is the most likely diagnosis?

A. AIDS dementia complex
B. CMV polyradiculopathy
C. Cryptococcal meningoencephalitis
D. Vacuolar (HIV) myelopathy
E. Zidovudine-related toxicity
Explanation:
The correct answer is
D. This is one of the most common neurologic complications of AIDS. Its pathologic substrate is degeneration of the spinal tracts in the posterior and lateral columns, which have a vacuolated microscopic appearance. Although the morphologic changes and clinical manifestations are similar to those associated with vitamin B12 deficiency, the pathogenetic mechanism is probably not related to dietary deficiencies. Since there is no specific clinical or laboratory test available for the diagnosis of this syndrome, vacuolar myelopathy in AIDS patients remains a diagnosis of exclusion. This implies that other HIV-related neurologic complications must be ruled out (see below).

AIDS dementia complex (choice A) manifests with progressive memory loss, alterations in fine motor control, urinary incontinence, and altered mental status.

CMV polyradiculopathy (choice B) may simulate HIV myelopathy and is a relatively frequent complication of AIDS. It can be excluded by the results of CSF analysis. CMV infection leads to neutrophilic pleocytosis in the CS
F.

Cryptococcal meningoencephalitis (choice C) would lead to signs and symptoms of meningitis. The CSF would show the fungal organism, which can be detected by special stains and culture studies.

Zidovudine-related toxicity (choice E) would lead to proximal muscle weakness and tenderness due mainly to a myopathic process.

9) A 45-year-old man is involved in an automobile accident and sustains severe injuries with considerable blood loss and hypotension. He is transferred from the emergency department to an intensive care unit, where he develops multiorgan failure. During the first 2 days in the intensive care unit, his plasma phosphate was within normal limits. Subsequently, it began to rise, eventually reaching 6.0 mg/dL. Failure of which of the following organs would most likely have this effect?
A. Heart
B. Kidneys
C. Liver
D. Lungs
E. Pancreas
Explanation:
The correct answer is
B. The usual cause of hyperphosphatemia is advanced renal insufficiency that destroys the kidneys' ability to excrete phosphate, thereby leading to hyperphosphatemia. Other causes include hypoparathyroidism, pseudohypoparathyroidism, and excessive oral phosphate administration. Acute transcellular shifts of phosphate into the extracellular space can also occur in the settings of diabetic ketoacidosis, crush injuries, rhabdomyolysis, systemic infections, and tumor lysis syndrome.

Failure of the heart (choice A), liver (choice C), or lungs (choice D) has no direct effect on phosphate metabolism.

Pancreatic failure (choice E) has no direct effect on phosphate metabolism. Diabetic ketoacidosis can cause a transcellular shift of phosphate into the extracellular space.

10) A 28-year-old woman is diagnosed with lupus nephritis, World Health Organization (WHO) type IV. She has a malar rash, diffuse arthritis, and edema. Her blood pressure is 190/110 mm Hg. Her creatinine is 2.1 mg/dL with a blood urea nitrogen of 28 mg/dL. Her urine reveals 25 red blood cells per hpf, and 3+ protein. One red blood cell cast is seen. A 24-hour urine collection reveals a protein of 11 grams with a creatinine of 1 gram. Which of the following would be the most appropriate management?
A. Oral azathioprine
B. Oral cyclophosphamide
C. Oral gold
D. Oral prednisone
E. Pulse IV cyclophosphamide

Explanation:
The correct answer is
E. The most effective treatment for aggressive systemic lupus erythematosus with nephritis is pulse cyclophosphamide. This has been shown to be the best agent to treat type IV lupus nephritis.

Oral cyclophosphamide (choice B) is effective but not as effective as the pulse IV form.

Oral gold (choice C) is used to treat rheumatoid arthritis.

Oral prednisone (choice D) and oral azathioprine (choice A) are other commonly used agents but are not as effective as cyclophosphamide.

11) A 48-year-old woman is seen by a clinician. She has a bright red, sharply demarcated, oozing and crusting rash involving one breast in the areola area. She has had this lesion for six months and states that it is slowly growing. The lesion does not respond to antibiotic ointment, antifungal ointment, or steroid ointment. This lesion is most likely related to which of the following conditions?
A. Breast cancer
B. Crohn disease
C. Gastric cancer
D. Rheumatoid arthritis
E. Systemic lupus erythematosus
Explanation:
The correct answer is
A. This is Paget disease of the breast, which actually corresponds microscopically to the presence of individual adenocarcinoma cells in the epidermis. When it involves the nipple area, it usually overlies an area of breast cancer. (Paget disease can also involve the vulva and vagina, where it may be unrelated to bulk cancer.) It is important to recognize the presentation, since focusing on treating the skin lesion empirically can lead to a delay in recognizing the cancer. The clinical description given in the question stem is typical. Mastectomy is a common form of treatment in these cases because of the nipple involvement.

Crohn disease (choice B) can cause erythema nodosum (deep subcutaneous nodules on lower legs).

Gastric cancer (choice C) can cause supraclavicular node enlargement due to metastasis.

Rheumatoid arthritis (choice D) can cause subcutaneous rheumatoid nodules.

Systemic lupus erythematosus (choice E) can cause a rash, most commonly seen on the face.

12) A recent article in a prominent medical journal explored the disparity of resource utilization between men and women. More men than women have major cardiac procedures, including catheterization, performed. This is thought to be because of which of the following reasons?
A. Fewer outcome studies have studied women with these disease
B. The incidence of cardiovascular disease is lower in women
C. Men have better health insurance
D. Men receive too many cardiovascular procedures
E. Provider attitudes has led to this situation
Explanation:
The correct answer is
E. Attitudes of physicians has been shown to have a significant effect on the delivery of health care. Health providers believe men to be more likely to have cardiac disease than women and this has guided practice, leading to a larger amount of cardiac procedures for men, compared with women.

Fewer outcome studies have studied women because of the belief that cardiac disease is more common in men (choice A).

The incidence of cardiovascular disease in women is similar to that in men (choice B) across the entire life-span.

There is a significant disparity between men and women in the use of cardiovascular interventions in the absence of financial (i.e., insurance) barriers (choice C).

One study has reported that women receive more appropriate cardiac services than men and that major interventions in men are over-utilized (choice D), but this does not explain the overall disparity.

13) A 43-year-old man presents with a 4-year history of joint pain. The distribution is asymmetric, involving the proximal and distal small joints of the right hand, the left knee, the ankle, and right elbow. Pain and morning stiffness are moderate. Physical examination reveals mild nail pitting, and the distal third interphalangeal joint is partially subluxated. X-rays of the hands show resorption of the distal end of the phalanx. The erythrocyte sedimentation rate (ESR) is elevated to 46 mm/hr, and rheumatoid factor is negative. Which of the following is the most likely diagnosis?
A. Primary generalized osteoarthritis
B. Pseudogout
C. Psoriatic arthritis
D. Rheumatoid arthritis
Explanation:
The correct answer is
C. Nail pitting and dystrophy associated with distal destructive asymmetric arthritis are virtually diagnostic of psoriatic arthritis. Skin disease may or may not be severe and obvious. Nail changes occur in 85% of those with psoriatic arthritis, and in only 20% of those with uncomplicated psoriasis. The clinical categories of psoriatic arthritis include distal interphalangeal, asymmetric, symmetric, mutilating, and spinal. Antimalarials should be avoided if disease-modifying therapy is indicated, as they can exacerbate psoriasis.

Primary generalized osteoarthritis (choice A) can involve the distal interphalangeal joints; joint erosions do not occur, and osteophytes are seen radiologically. Nail dystrophy does not occur.

A variant of pseudogout (choice B) can closely mimic rheumatoid arthritis or a mutilating arthropathy, but nail changes are absent, and there is radiologic evidence of chondrocalcinosis (calcification of articular cartilage).

Rheumatoid arthritis (choice D) does not cause distal erosive disease and is generally (but not invariably) symmetric.

14) A 45-year-old man presents to a physician with complaints of weakness, fatigue, and feeling near fainting when he stands up quickly. Screening chemistry studies demonstrate sodium, 128 mEq/L; potassium, 5.2 mEq/L; bicarbonate, 17 mEq/L; and urea nitrogen, 45 mg/dL. The physician is considering Addison disease in his differential diagnosis. Which of the following features on physical examination would be most suggestive of this diagnosis?
A. Black freckles on the shoulders
B. Large, furrowed tongue
C. Many spider angiomas
D. Protruding eyeballs
E. Small glistening bumps on the lips
Explanation:
The correct answer is
A. While there is some variation in the usage of the term, Addison disease is usually taken to mean adrenocortical insufficiency related to disease that destroys the adrenal gland. Most authors separate out secondary adrenocortical insufficiency due to pituitary failure and recent or current exogenous steroid therapy. True Addison disease, which is not related to inadequate pituitary secretion of ACTH, frequently has stigmata of hyperpigmentation relating to a melanocyte-stimulating hormone (MSH) effect seen with high ACTH levels. The biochemical basis of this is a homology between part of the ACTH molecule and the MSH molecule. Typical hyperpigmentation features include black freckles of the shoulders, head, and neck; bluish-black discoloration of areolas and mucous membranes (both oral and anogenital); and diffuse tanning, specifically including non-sun-exposed skin. The pattern of laboratory screening studies illustrated in the question stem is also very suggestive, with very low serum sodium, high potassium, low bicarbonate, and high serum urea nitrogen.

A large, furrowed tongue (choice B) suggests acromegaly.

Many spider angiomas (choice C) suggest chronic liver disease.

Protruding eyeballs (choice D) suggest Grave's disease.

Small glistening bumps on the lips (choice E) suggests the mucosal neuromas of MEN IIb.

15) A 71-year-old man presents to the emergency department with fever and cough. He has known hypercholesterolemia and is status post a right hemicolectomy for colon cancer. The patient states that he has had 3 days of fever to 102 F, cough productive of green sputum, as well as general malaise and weakness. His physical examination is remarkable for decreased breath sounds at the left base, left basilar egophony and dullness to percussion. A complete blood count reveals a leukocyte count of 15000/mm3. A chest radiograph reveals a left lower lobe infiltrate. Which of the following is the most important part of the history to ascertain prior to initiating therapy?
A. Influenza immunization status
B. Plasma lipid profile
C. Stage of the colon cancer
D. Social support structure for home therapy
E. Recently hospitalizations
Explanation:
The correct answer is
E. This patient clearly has pneumonia. The absolute requirement to diagnose pneumonia is an infiltrate on chest radiograph, coupled with clinical findings suggestive of a pneumonia, which this patient has. The next relevant issue is, what is the likely organism? If this man were living at home, the most common organism is the pneumococcus. However, given his recent hemicolectomy, the possibility of a more virulent organism emerges. This is the most critical factor in dictating his course of therapy.

Whether the patient received the annual influenza vaccine (choice A) is important given his age and the morbidity associated with an influenza infection, however, it has no bearing on the course of this pneumonia or its therapy.

His lipid status (choice B) is not relevant to therapy of his pneumonia.

The stage of his colon cancer (choice C) is important, but not for the treatment of his pneumonia. There is no clinical relationship between pneumonia severity and colon cancer progression until late stage metastases occur and there is a possibility of post-obstructive pneumonia developing.

The patients social support structure for home therapy (choice D) is an issue only at the time of discharge when the issue of continued home therapy is important. At this time his ability to have assistance with possible intravenous medications is an issue.

16) A 49-year-old woman presents to the office because of complaints of fatigue. She has had progressive exercise intolerance over the prior 6 months. On physical examination, she is pale and afebrile. Her blood pressure is 112/68 mm Hg, and her pulse is 88/min. Heart and lung examinations are normal except for a I/VI systolic flow murmur at the left sternal border. Routine laboratory results reveal hemoglobin of 8.3 g/dL, a mean corpuscular volume of 118 µL/m3, and a B12 of 82 pg/mL (normal >210 pg/mL). She undergoes a Schilling test, which reveals malabsorption of radiolabeled B12. Intrinsic factor is administered and the radiolabeled B12 is subsequently absorbed. Which of the following is the diagnosis?
A. Atrophic gastritis
B. Bacterial overgrowth
C. Chronic pancreatitis
D. Crohn disease
E. Gastric ulcer
Explanation:
The correct answer is
A. This patient has pernicious anemia, as demonstrated by correction of her deficiency in intrinsic factor production by her parietal cells. This is an autoimmune disease directed against the parietal cells of the stomach, which are the normal producers of the intrinsic factor needed for absorption of vitamin B12. The lack of B12 then causes development of a megaloblastic (with high mean corpuscular volume) anemia. Biopsy of the gastric mucosa in these cases reveals atrophic gastritis. The Schilling test examines the absorption of radioactively labeled vitamin B12 before and after administration of intrinsic factor. A result of poor absorption of B12 before administration of intrinsic factor and good absorption after strongly suggests pernicious anemia as the diagnosis. Patients with pernicious anemia require long-term (probably life-long) parenteral replacement of vitamin B12 and may also have other significant autoantibodies, notably those directed against thyroid antigens.

Choices B, C, and D may all produce a B12 deficiency, but they do not correct with intrinsic factor. In patients with bacterial overgrowth (choice B), the excess bacteria will preferentially absorb intraluminal B12.

Chronic pancreatitis (choice C) may predispose to a B12 deficiency by failing to secrete the enzymes that are necessary to cleave the salivary R factor from the B12, making it unavailable for binding to intrinsic factor.

Crohn disease (choice D) may cause a B12 deficiency if the terminal ileum is severely inflamed or has been resected.

Gastric ulcer (choice E) can cause anemia secondary to bleeding, but it would be a normocytic anemia (unless there had been enough blood loss to cause iron deficiency, in which case it would microcytic) and would not be expected to respond to B12.

17) A 65-year-old woman consults a physician because of a 3-month history of weight loss, burning sensation of the tongue, fatigue, anorexia, and poorly localized abdominal pain. The woman appears pale to the physician. Intraoffice hematocrit is 35% with peripheral smear showing large erythrocytes and hypersegmented neutrophils. Serum folate is 2.4 ng/mL (normal greater that 1.9 ng/mL) and serum vitamin B12 is 100 pg/mL (normal 200-800 pg/mL). Stomach biopsy demonstrates chronic gastritis. Autoantibodies to which of the following are most likely involved in this patient's condition?
A. Basement membrane
B. Insulin receptor
C. Intrinsic factor
D. SS-B
E. TSH receptor
Explanation:
The correct answer is
C. This patient has pernicious anemia, in which autoimmune gastritis causes a lack of the intrinsic factor needed to absorb vitamin B12. Autoantibodies that are often present include those directed against the microsomal fraction of parietal cells and those capable of neutralizing intrinsic factor. The result is that vitamin B12 can no longer be absorbed by the terminal ileum. Since some vitamin B12 is stored in the liver, deficiency tends to develop slowly. Vitamin B12 deficiency can cause megaloblastic anemia; neurologic abnormalities that tend to begin with loss of position and vibration sense; and GI manifestations including anorexia, intermittent constipation or diarrhea, and abdominal pain.

Antibodies to basement membrane (choice A) are associated with Goodpasture syndrome.

Antibodies to insulin receptors (choice B) are associated with insulin resistance.

Antibodies to SS-B (choice D) occur in association with Sjögren syndrome.

Antibodies to TSH receptor (choice E) are associated with Graves disease.

18) A previously healthy 50-year-old woman comes to the physician because of double vision for three days. Her temperature is 37 C ([snip].6 F). The patient denies nausea or vomiting. Examination reveals ptosis and slight divergence of the right eye. Extraocular movements are limited in all directions, except laterally. The right pupil is larger than the left and poorly reactive to light. Examination of the fundus fails to reveal papilledema. Which of the following is the most likely underlying condition?
A. Aneurysm of the posterior communicating artery
B. Carcinoma of the right pulmonary apex
C. Diabetes mellitus
D. Giant cell arteritis
E. Syphilis
F. Systemic hypertension
Explanation:
The correct answer is
A. This patient displays signs of oculomotor palsy, with restriction of the eye movements in all directions (except laterally, due to preservation of the sixth cranial nerve, the abducens), and ptosis. Dilatation of the pupil, which fails to react to light, is a sign of intracranial compression of the third, or oculomotor cranial nerve. This should prompt search for an underlying surgical cause of oculomotor palsy. Uncal herniation and aneurysm of the posterior communicating artery are the two most common surgical conditions leading to oculomotor palsy. In the absence of clinical evidence of increased intracranial pressure, it may be assumed that the patient has an aneurysm of the posterior communicating artery until proven otherwise. Cerebral angiography is the investigation of choice to confirm the diagnosis.

All of the most common medical causes of oculomotor nerve palsy result in paresis of extraocular movements and ptosis, but the pupillary light reflex is preserved. These conditions include diabetes mellitus (choice C), giant cell arteritis (choice D), syphilis (choice E) and systemic hypertension (choice F).

Carcinoma of the right pulmonary apex (choice B) may result in Horner syndrome (miosis, ptosis, enophthalmos, and loss of sweating on the affected hemiface) due to infiltration of the cervical autonomic ganglia.



19) A patient with a history of chronic bacterial sinusitis presents to the emergency department with a very severe headache. While waiting to be seen, he develops a generalized grand mal seizure. Physical examination, after the seizure is over, demonstrates high fever, exophthalmos, papilledema, and nerve palsies of the VI and III cranial nerves on one side. Which of the following is the most appropriate next step?
A. Admit to the medical floor for monitoring of progression of symptoms
B. Emergency CT scan
C. Emergency exploratory surgery
D. Emergency ultrasound
E. Keep in emergency department for monitoring of progression of symptoms
Explanation:
The correct answer is
B. This is the way that cavernous sinus thrombosis presents. This condition is due to a septic thrombosis that can complicate chronic bacterial sinusitis. Meningitis is another significant possibility. Lumbar puncture is dangerous in a patient with increased intracranial pressure, as indicated by the papilledema. Emergency CT scan of the cavernous sinus, air sinuses, orbit, and brain is warranted. Additionally, cultures of blood and any nasal discharge are warranted; Gram's stain of the nasal discharge may give a preliminary indication of the causative organism. High dose intravenous antibiotics are started, and then altered, if necessary, when culture results are reported. Cavernous sinus thrombosis has a 30% mortality rate, even when prompt, appropriate medical care is given.

Simply monitoring (choices A and E) a patient like this would be very dangerous. Ultrasound (choice D) would probably not adequately visualize the complex structures of the sinuses, orbits, and brain.

Surgery (choice C) is not indicated in this setting.

20) A 41-year-old man presents with complaints of mild intermittent heartburn after meals for the past 6 months. He has tried various over-the-counter antacids and H2 receptor antagonists with only minimal relief. He denies any dysphagia or odynophagia, and is otherwise in good health. He is concerned about the risk of developing cancer, because his father died of gastric cancer at age 49. His physical examination is unremarkable. Which of the following would be the most appropriate next step in management?
A. Avoidance of a high-protein diet
B. Avoidance of aspirin
C. Avoidance of acetaminophen
D. Elevation of the head of his bed
E. Increased consumption of carbohydrates
Explanation:
The correct answer is
D. Before initiating pharmacologic therapy, it is worthwhile to consider lifestyle modifications that may reduce symptoms in patients with gastroesophageal reflux disease (GERD). In this regard, elevating the head of his bed is an important step, since it will reduce the degree of nocturnal acid reflux while the patient is in the supine position. Other nonpharmacologic measures that may be helpful include avoidance of strong stimulants of acid secretion (coffee, alcohol), avoidance of certain drugs (anticholinergics) and foods (fats, chocolates), and cessation of smoking.

Although fatty foods may exacerbate GERD by reducing the pressure on the lower esophageal sphincter (LES), high-protein diets (choice A) and carbohydrates (choice E) have no particular effect on the mechanism or symptoms or GER
D.

Although aspirin (choice B) may be injurious to the gastric and duodenal mucosa, it is not implicated in exacerbations of GER
D.

Acetaminophen (choice C) has no effect on the symptoms or cause of GER
D.

21) A 40-year-old woman is brought to the emergency department following a suicide attempt with imipramine. Her fiancee found her unresponsive, with an empty bottle of the imipramine at her side. The imipramine had been his, and the prescription had been filled that morning. Her past medical history is significant for hypertension, atrial fibrillation, diabetes, and asthma. Her medications include furosemide, procainamide, glyburide, prednisone, and albuterol. She has no known drug allergies. She is afebrile, has a blood pressure of 100/60 mm Hg, pulse of 62/min, and respirations of 22/min. A gastric lavage yields multiple pill fragments. She is confused and somnolent, and has shallow respirations. Her physical examination is otherwise unremarkable. On an ECG, which of the following abnormalities would most likely reflect possible cardiac toxicity?
A. Left deviation of the QRS axis
B. Prolongation of the QRS interval
C. Shortening of the QT interval
D. ST segment depression
E. T wave inversion
Explanation:
The correct answer is
B. A prolongation of the QRS interval is highly predictive of both cardiac and CNS toxicities from tricyclic antidepressant ingestion.

Left deviation of the QRS axis (choice A), which can be seen with conditions such as left ventricular hypertrophy and left bundle branch block, is not typically associated with tricyclic cardiac toxicity. Conversely, right deviation of the QRS axis (greater than 120 degrees) is very predictive of cardiac toxicity from tricyclics.

Shortening of the QT interval (choice C) is not seen with tricyclic toxicity, but can be seen with metabolic derangements such as hypercalcemia.

Neither ST segment depression (choice D) nor T wave inversion (choice E) is directly associated with cardiac toxicity from tricyclic overdose. These changes may be seen, however, in conjunction with the more classic ECG manifestations of tricyclic toxicity (prolonged QRS interval, right axis deviation) if the resulting cardiac toxicity leads to diminished coronary perfusion and ischemia.

22) A 43-year-old bus driver presents to his gastroenterologist with complaints of difficulty swallowing solid foods. The evaluation demonstrates a smooth, tapered stricture of the distal esophagus, and biopsies reveal changes consistent with chronic esophagitis and fibrosis. The stricture is dilated with an endoscopic balloon dilator, and the patient's symptoms resolve. He reports that although he has had dysphagia for the past 2 months prior to the endoscopy, he rarely has heartburn and uses an over-the-counter antacid only occasionally. Which of the following is the most appropriate future management of this patient?
A. Famotidine
B. Lansoprazole
C. Magnesium hydroxide
D. Metoclopramide
E. No medication is necessary
Explanation:
The correct answer is
B. Although this patient has rarely been aware of symptoms of gastroesophageal reflux disease (GERD), the development of a peptic stricture clearly indicates longstanding acid reflux into the distal esophagus. This will be a persistent process and, if not treated, will lead to recurrent strictures. He therefore requires chronic treatment with a proton pump inhibitor to suppress acid secretions.

Famotidine and antacids, such as magnesium hydroxide (choices A and C), are adjuncts to the mainstay of therapy, which is proton pump inhibition.

Even though the patient is not symptomatic, he does require continued acid suppression. Metoclopramide (choice D) reduces the lower esophageal sphincter pressure and is an adjunct to acid suppression in the management of patients with reflux. It is not used as first-line therapy, however. It is nowhere near as effective as proton pump inhibitors and frequently leads to side effects of sedation because of its ability to cross the blood-brain barrier and inhibit dopamine, producing Parkinson-like symptoms.

As stated above, this patient will have recurrent strictures if he does not receive treatment. Therefore, choice E is incorrect.

23) A 23-year-old professional basketball player presents to the team physician 3 hours before game time complaining of abdominal pain. The symptoms began approximately 8 hours earlier in a diffuse fashion. Two hours later, he began feeling nauseated and vomited twice. Over the past 4 hours, the abdominal pain has become more severe and well localized in the right lower quadrant. His examination now reveals well-localized pain in the right lower quadrant inferolateral to the umbilicus. Which of the following is the most likely diagnosis?
A. Acute obstruction of the appendiceal lumen by a fecalith
B. Acute onset of ileocolitis
C. Acute onset of ischemic colitis
D. Acute Yersinia infection
E. Obstruction of the ileocecal valve by a mass
Explanation:
The correct answer is
A. Acute appendicitis is the second most common cause in the U.S., behind hernia, of severe acute abdominal pain that requires abdominal operation. Although it can occur at all ages, many patients, like this man, are teenagers or young adults. This patient's presentation is typical for acute appendicitis, with initially poorly localized pain that is followed by nausea and vomiting. In classic appendicitis, the pain shifts to the right lower quadrant, where it becomes more localized. In most patients, acute obstruction of the appendiceal orifice by a fecalith initiates the acute appendicitis.

The acute onset of ileocolitis (choice B) will produce diarrhea or bloody stools.

There is no evidence to suggest an etiology for ischemic colitis (choice C), which will typically present with bloody diarrhea and often with left-sided abdominal pain.

Acute Yersinia infection (choice D) will produce acute right lower quadrant findings similar to those of acute appendicitis. However, it is accompanied by diarrhea, which is not described in this case.

There is no reason to suspect obstruction of the ileocecal area by any mass (choice E) in a 23-year-old man. Such an obstruction, should it occur, would typically present with abdominal distention as a result of bowel obstruction.

24) A 31-year-old homeless woman is brought to the emergency department after being found face down on the street. The woman has a long history of admissions to the hospital for alcohol-related issues, including seizures, withdrawal, and hallucinations. Today, she was seen to fall in the street, have what were described as "convulsions" and then vomit. She remained face down in the street until the paramedics arrived. On physical examination, she has dry mucous membranes, a jugular venous pressure of less than 5 cm, and diffuse ecchymoses on her face, body, and breasts. Which of the following vitamins should be administered prior to volume resuscitation with IV fluids containing glucose?
A. Vitamin B1 (Thiamin)
B. Vitamin B3 (Niacin)
C. Vitamin B12 (Cobalamin)
D. Vitamin C
E. Vitamin K
Explanation:
The correct answer is
A. Administering glucose to a patient who is deficient in thiamin may precipitate Wernicke-Korsakoff syndrome, which is a combination of confusion, ataxia, ophthalmoplegia, anterograde and retrograde amnesia, and confabulation. It is therefore imperative to administer IV thiamin prior to glucose-containing IV fluids.

Niacin (choice B) is an essential component of the coenzymes involved in oxidation-reduction reactions. Profound deficiency in niacin causes the classic triad of pellagra: dermatitis, diarrhea, and dementia.

Vitamin B12 deficiency (choice C) may lead to megaloblastic anemia, neurologic complications, and dementia.

Vitamin C deficiency (choice D) may lead to difficulty with wound healing and scurvy.

Vitamin K (choice E) is essential for the production of selected clotting factors. Although alcoholics may be deficient in all the vitamins mentioned in the answer choices, only deficits in thiamin are associated with harmful effects if glucose is administered without replenishment.

25) A 36-year-old man develops rapid mental status deterioration two days after sustaining a femoral fracture in a skiing accident. Physical examination shows multiple petechiae in the anterior chest and abdomen. On the third day, the patient lapses into coma and dies. Postmortem examination of the brain reveals numerous petechial hemorrhages in the corpus callosum and centrum semiovale. Which of the following is the most likely diagnosis?
A. Diffuse axonal injury
B. Fat embolism
C. Septic embolism
D. Systemic thromboembolism
E. Watershed infarction
Explanation:
The correct answer is
B. The clinical manifestations are consistent with fat embolism. This complication is frequent, following fractures of long bones, but is usually asymptomatic. Fat embolism mainly affects the lungs and the brain, and the clinical picture consists of dyspnea, tachycardia, and mental status changes. Only rarely, does this condition lead to death. In the lungs, fat emboli can be visualized histologically. In the brain, multifocal petechiae in the white matter represent the most common pathologic change.

Diffuse axonal injury (choice A) is one of the most common forms of traumatic brain injury. It involves the central white matter, especially the corpus callosum and cerebral peduncles. It is sometimes associated with small petechiae in these areas. The patient may develop coma a few hours to days after head trauma.

Septic embolism (choice C) results from septic emboli lodging in the terminal intraparenchymal arteries of the brain. It leads to multiple cortical infarcts, usually of the hemorrhagic type. The white matter is spared.

Systemic thromboembolism (choice D) is usually of cardiac origin—for example, in patients with cardiac arrhythmias with thrombi in the right atrium or ventricle. Thromboemboli in the brain cause hemorrhagic infarction in the cortex.

Watershed infarction (choice E) is often seen in patients suffering from acute hypotensive episodes, especially if the circle of Willis is already compromised by atherosclerotic change. The cortical regions at the border zone between different vascular territories (e.g., between the distribution of the anterior and middle cerebral arteries) undergo ischemic necrosis.

26) A 45-year-old man consults a physician because of dysuria. The patient is treated with antibiotics, but symptoms recur one week after antibiotic therapy is stopped. A different antibiotic is tried, but symptoms again recur after cessation of the antibiotic. Rectal examination demonstrates an enlarged prostate with areas of tenderness and fluctuance. Which of the following is the most likely diagnosis?
A. Benign prostatic hyperplasia
B. Chronic nonbacterial prostatitis
C. Prostadynia
D. Prostatic abscess
E. Prostatic carcinoma
Explanation:
The correct answer is
D. The patient has a prostatic abscess. The typical age is 40 to 60 years, and is consequently somewhat younger than the ages at which benign prostatic hyperplasia and prostate cancer become major problems. Infecting organisms include aerobic gram-negative bacilli and Staphylococcus aureus. Prostatic abscess should be suspected when a man develops repeated urinary tract infections that seem to get better with antibiotic therapy, only to recur later. The most important diagnostic clue, if detectable, is the presence of a fluctuant mass in the prostate on rectal exam. Some patients have only prostatic enlargement, or even no positive findings on physical examination. Patients may have normal urine, although it is more usual for an organism to be cultured at some point. Prostatic ultrasound may be helpful if the diagnosis is suspected. A few cases are even picked up at the time of prostatic resection for benign prostatic hyperplasia or other disease. Treatment is with evacuation of the abscess by a transurethral or perineal route followed by appropriate antibiotics.

Benign prostatic hyperplasia (choice A) can cause urinary obstruction predisposing for bladder infection, but the prostate would not be fluctuant.

Chronic nonbacterial prostatitis (choice B) can cause symptoms resembling urinary tract infection, but would not cause a fluctuant prostate.

Prostadynia (choice C) is a noninfectious, noninflammatory condition of younger men that can mimic prostatitis, but would not cause a fluctuant prostate.

Prostatic carcinoma (choice E) is usually asymptomatic, and can cause a firm prostatic mass.

27) A 31-year-old woman presents at the hospital for a pre-employment physical examination prior to beginning her year as a medical intern. She is sexually inactive and denies alcohol use. She had infectious mononucleosis while in college and received the recombinant hepatitis B vaccine before starting medical school. Which of the following would describe her hepatitis B serologic profile?
A. Hepatitis B surface antigen positive, core antibody positive, and surface antibody negative
B. Hepatitis B surface antigen negative, core antibody positive, and surface antibody positive
C. Hepatitis B surface antigen positive, core antibody negative, and surface antibody negative
D. Hepatitis B surface antigen negative, core antibody negative, and surface antibody positive
E. Hepatitis B surface antigen negative, core antibody negative, and surface antibody negative.
Explanation:
The correct answer is
D. Patients who receive the hepatitis vaccine will develop only surface antibodies, since the vaccine contains only epitopes of the surface antigen and NOT of the intact viral particle, which contains the core antibody. These patients will therefore have a negative surface antigen, negative core antibody, and positive surface antibody.

Choice A describes a patient who has recent hepatitis B infection and has not yet developed surface antibody
B.

Choice B describes a patient that has had a past hepatitis B infection and has developed immunity.

Choice C refers to a patient who has developed acute hepatitis B and has not yet developed immunity.

Choice E describes a patient who has never been exposed to surface antigen and has no immunity.

28) A 57-year-old man presents to his physician for a preoperative evaluation. He has been a long-time patient in this office and has been treated for hypertension and gastritis. He has been scheduled for an elective open cholecystectomy in 2 days. He currently takes omeprazole for his gastritis and thiazide for his hypertension daily. He smokes two packs of cigarettes per day. His home blood pressure log shows that his systolic pressures range from 150 to 190 mm Hg, and his diastolic pressures range from 80 to 105 mm Hg, indicating that his blood pressure may be not adequately controlled for the surgical procedure. Which of the following medications is most appropriate in the perioperative period for added blood pressure control?
A. Captopril
B. Clonidine
C. Metoprolol
D. Nifedipine
E. Prazosin
Explanation:
The correct answer is
C. There is an extensive body of literature indicating that beta blockers given to non-cardiac surgical patients who are at risk of cardiac events are associated with a more favorable outcome in terms of postoperative cardiovascular morbidity and mortality. This patient has somewhat poorly controlled hypertension, as well as at least three cardiovascular risk factors (hypertension, tobacco, age). Ideally, one would like to have better control of the blood pressure and to reduce any risk for adverse perioperative events. Beta blockers can achieve both of these endpoints.

Captopril (choice A) is an ACE inhibitor that has good efficacy in the treatment of hypertension. This class of drugs has also been shown to prolong survival in patients with congestive heart failure.

Clonidine (choice B) is a central alpha-2 receptor agonist that works to attenuate sympathetic outflow and thus lower blood pressure. Although it is a reasonably efficacious drug, it is associated with rebound hypertension if abruptly discontinued. It has no role in the perioperative management of blood pressure.

Nifedipine (choice D) is a calcium channel blocker that has reasonable efficacy in treating hypertension. There is no benefit to giving this agent in the perioperative period.

Prazosin (choice E) is a alpha-1 receptor antagonist that is very efficacious in the treatment of hypertension. This class of drugs is also useful in the treatment of benign prostatic hypertrophy (BPH).

29) A 66-year-old man presents to the clinic complaining of progressively worsening shortness of breath and nonproductive cough over the past 2 years. He retired 1 year ago, after working as a rock miner for more than 30 years. He has no other significant past medical history. On physical examination, he is a thin man who appears tachypneic at rest. His lungs have reduced chest expansion and dry inspiratory rales in the upper lobes bilaterally. The remainder of his examination is normal. A chest x-ray film reveals multiple round opacities in the upper lobes accompanied by hilar lymphadenopathy with lymph node calcification. Which of the following is the most likely diagnosis?
A. Asbestosis
B. Aspergillosis
C. Cystic fibrosis
D. Silicosis
E. Tuberculosis
Explanation:
The correct answer is
D. This patient's occupational history of working in the mining industry should always prompt the consideration of the diagnosis of silicosis. There is usually bilateral upper lobe involvement associated with hilar lymphadenopathy and "eggshell" calcification of the visualized lymph nodes.

There is no description of asbestos exposure (choice A), making this diagnosis unlikely.

Bronchopulmonary aspergillosis (choice B) is often seen in farm workers who have been working in silos.

Cystic fibrosis (choice C) presents during childhood, and these patients do not survive to this age.

There is no evidence of tuberculosis exposure (choice E) by the patient's history.

30) A 55-year-old woman with a long-standing history of atrial fibrillation secondary to mitral regurgitation presents to the emergency department with a painful right foot. The patient reports that, over the past few hours, her foot has become more painful and now is nearly insensate. She describes the pain as burning and states that it is not relieved by any intervention. She takes coumadin, atenolol, digoxin, and aspirin. On physical examination, her pulse is irregularly irregular. Her lungs are clear, and she has a loud holosystolic murmur heard best at the apex. Her right foot is gray and cool to the touch and has poor capillary refill. Dorsalis pedis and posterior tibial pulses are absent on the right. Her prothrombin time is 14.4 seconds (INR 1.4). Which of the following is the most appropriate course of action?
A. Arrange for her to be seen by a vascular surgeon in the emergency department now
B. Arrange for her to be seen by a neurologist within the next few days
C. Arrange for her to undergo an MRI of the head that day
D. Ask her to make an appointment to be in seen in your office within 1 week
E. Instruct her to soak her leg in warm water and to place a fitted stocking on her affected leg
Explanation:
The correct answer is
A. The symptoms and signs that she is describing, particularly in the context of atrial fibrillation (AF), suggest peripheral embolization, which is a surgical emergency. The treatment of choice involves immediate embolectomy, which is usually performed by a vascular surgeon, followed by anticoagulation. Her subtherapeutic prothrombin time and persistent AF on examination are supportive of this diagnosis.

All the other choices (choices B through E) represent actions that would delay surgical care and likely lead to the loss of her limb. Even if the physician has not seen this patient before, it is incumbent on him to direct her to and facilitate immediate surgical intervention.

31) A 50-year-old man is brought to the emergency department complaining of light-headedness. He has a history of lung cancer, which was diagnosed a month ago and found to be widely metastatic to the bone and pericardium. On physical examination, his blood pressure is 70/40 mm Hg, and his pulse is 100/min. His heart sounds are distant and soft. His ECG demonstrates low voltage, and electrical alternans is present. A chest x-ray film shows that the cardiac silhouette has a "water bottle" appearance.Which of the following is the most appropriate intervention in this patient?
A. Beta-blockers
B. Nonsteroidal anti-inflammatory drugs
C. Steroids
D. Pericardiocentesis
E. Cardiac catheterization
Explanation:
The correct answer is
D. This patient is in pericardial tamponade, most probably as a result of his malignancy. Lung cancer is particularly likely to cause pericardial effusions. Furthermore, since this patient has metastases to the pericardium, he might be bleeding into the pericardial space. This tamponade may be the cause of his significant hypotension and the soft cardiac sounds. Electrical alternans, a phenomenon in which the QRS changes axis, is indicative of pericardial effusion, since the heart is moving freely in the fluid, causing a change in axis noted on the ECG. Emergently, this patient needs decompression of the pericardial space with the aid of pericardiocentesis, whereby a catheter directly drains the fluid in the pericardial sac.

Beta-blockers (choice A) would be of no benefit in treating cardiac tamponade.

Nonsteroidal anti-inflammatory drugs (NSAIDs) (choice B) can be useful in treating pericarditis, which may cause pericardial effusions. However, this is a longer term option and will have little utility emergently.

Steroids (choice C) may similarly be used in pericarditis, after NSAIDs have failed. However, this is an option to be explored after the pericardial fluid has been drained.

Cardiac catheterization is often used to confirm the diagnosis of tamponade (choice E). Typically the pressure equalizes across the right atrium and ventricle. However, emergently, this patient should have pericardiocentesis.

32) A 62-year-old man with a 110 pack-year history of smoking presents with chest pain. He states that for the past few months, he has been getting chest "pressure" localized to the substernal region, radiating to the left arm on occasion. The pain occurs with mild exertion, but never at rest. He further states that when he gets the pain, it usually last about 5 minutes but goes away with rest. He reports that his exercise tolerance is moderate, and he gets dyspnea on exertion after a few blocks of walking. On physical examination, he has no chest wall tenderness to palpation, but a carotid bruit is heard, and his dorsalis pedis pulses are decreased. He has no history of coronary disease but his family history is significant for his father having a myocardial infarction at age 56. He denies chest pain at this time. In addition to ascertaining his other coronary risk factors, which of the following is the most appropriate diagnostic intervention?
A. Obtain a resting electrocardiogram
B. Schedule the patient for a cardiac echocardiogram
C. Schedule the patient for an exercise treadmill test
D. Schedule the patient for non-urgent coronary angiography
E. Schedule the patient for immediate coronary angiography
Explanation:
The correct answer is
C. This is a patient who has 3 clear risk factors for coronary artery disease (tobacco, family history and age) and based on his physical examination, likely has severe peripheral vascular disease. He has, by definition, typical chest pain, so called "new onset angina". He is a prime patient to have significant coronary disease, and thus we suspect ischemia as a cause for his pain. As a surrogate for coronary angiography, which actually shows anatomy, an exercise treadmill test allows us to detect ECG changes of ischemia with activity and thus stratify this patient as requiring intervention (such as percutaneous transluminal coronary angioplasty, or coronary artery bypass grafts), or perhaps angiography to better evaluate his anatomy.

A resting ECG (choice A) is appropriate, but not the most appropriate, given that he is pain-free at present and one would not expect to see any ECG changes associated with ischemia.

A cardiac echocardiogram (choice B) will likely be performed, given his dyspnea on exertion, but is not an appropriate test in the triaging of suspected ischemic chest pain. In some centers a "stress-echo", specifically a dobutamine echocardiogram, is used to evaluate ischemic potential.

A non-urgent coronary angiography (choice D) is also inappropriate since angiography is an invasive procedure reserved for people that have had equivocal results from less invasive diagnostic procedures, or are having signs of crescendo angina. This patient has new angina, but it is "typical" angina in that it is exertional.

An immediate coronary angiogram (choice E) is clearly not indicated as the patient is not having active ischemia or a myocardial infarction requiring reperfusion.

33) A 35-year-old man comes to the physician for a health maintenance examination. He received blood transfusions for hypovolemic shock following a gunshot wound 10 years earlier. He is currently in good health, and physical examination is unremarkable. A serum chemistry panel shows:
ALT 250 U/L
AST 140 U/L
Alkaline phosphatase 70 U/L
Serologic evaluation for viral hepatitis reveals positive antibodies to hepatitis C virus (HCV). A percutaneous liver biopsy shows marked portal inflammatory infiltrate disrupting the limiting plate of hepatic lobules. Which of the following is the incidence rate of this complication following HCV infection?
A. 5%
B. 10%
C. 20%
D. 40%
E. 80%
Explanation:
The correct answer is
E. The acute infection due to hepatitis C virus (HCV) is most commonly asymptomatic, but 80% of these cases progress to chronic hepatitis. Of the 80%, 20% will eventually evolve to cirrhosis. The source of infection remains unknown in a substantial number of cases, but 50% are related to IV drug abuse and 4% are attributable to blood transfusion. HCV, on the other hand, is now the most common cause of transfusion-associated hepatitis. The mode of presentation of chronic hepatitis C is often insidious, and patients might well be in good health when elevated aminotransferases are discovered. This laboratory finding prompts additional investigations, usually including a percutaneous liver biopsy. This will demonstrate the typical histologic changes of chronic hepatitis, namely chronic portal inflammation eroding, to varying extents, into the hepatic lobule. The degree of lobular "invasion" by the portal inflammatory infiltrate is the main indicator of the propensity for evolution to cirrhosis. Male sex, infection after age 40, and alcohol consumption are risk factors for evolution of chronic hepatitis C to cirrhosis. Nowadays, HCV is considered the most common cause of chronic hepatitis and one of the most common causes of cirrhosis in industrialized countries.

34) A 71-year-old woman is admitted to the hospital for pneumonia. The patient presented to the hospital 2 days ago for cough and fever. She reporte
Related Posts Plugin for WordPress, Blogger...